You are on page 1of 65

MT301B – Clinical Chemistry 2 (Lecture) • **N → Rheumatoid Arthritis (abnormal immune

system)
January 25, 2023 – Introduction & Recall
5) What is the clinical significance of an elevated of
purine metabolism? Gout
6) What is the test that represents muscle
PRE-TEST metabolism? Creatinine
7) What is the Standard Deviation (SD) that needs to
Part 1
be followed and applied to maintain Quality Control
(QC)? ± 2SD
1) What is the preferred specimen of choice for almost 8) In Quality Control, what is the term implies for
all Clinical Chemistry test profile? Serum closeness of test results to the true or mean value?
• **Not blood dahil general siya. Accuracy
• **Serum: 9) What is the characteristic of the serum sample that
o Cellular – packed RBC must be avoided in most Clinical Chemistry
o Non-cellular – liquid portion procedures? Hemolyzed
▪ Serum – clotted • **3 Characteristics of Serum Sample:
▪ Plasma – unclotted o Normal or Straw
2) Among the test in CC, which test needs to fast for o Chylous or Milky
12-14 hours? Triglyceride o Hemolyzed
3) What is the test requested for a patient with a 10) If the specimen received has a chylous
Diabetes Mellitus to check for a therapeutic characteristic, what profile test will be expected to
monitoring? HBA1C elevate? Lipid
• **Tests: 11) What anticoagulant is considered the naturally
o FBS – check once a week or once a month occurring anticoagulant? Heparin
o RBS • **Heparin (Green) – produces Basophils (may
o OGTT – gestational diabetes granules)
▪ FBS Yung 12) If red top is not available for FBS, what is the other
▪ Oral glucose (70-75 mL) iniinom color of the anticoagulated tube that can be used?
▪ 30 mins, 1 hour, 1.5 hour, 2 hours Gray
(2-hr PPBS) • **Gray – Flouride “GF” → has anticoaglycolytic
▪ 5 extraction 13) What is the other term for hemolyzed sample?
o OGCT – no fasting (RBS) Lake Sena
▪ Oral glucose 14) How long should the tourniquet be applied in the
▪ After an hour → TEST arm for venipuncture procedure? <1 minute or
o HBA1C within 1 minute
▪ TDM → 2-3 months 15) This term implies for a bluish-black discoloration of
▪ To check good prognosis ng drug – the skin due to a traumatic puncture. Hematoma
if nacocontrol ba ng drug ang blood
sugar; if hindi, ↑ dose of drug
▪ Tube: EDTA (examine whole blood Part 2
– packed RBC) 1) What is the preferred serum characteristic of
• **Lipid profile test: (LDL and VLDL → computed) choice for almost all Clinical Chemistry test profile?
o Cholesterol Straw Serum
o Triglyceride 2) What is the preferred specimen for bilirubin
o HDL/LDL determination? Serum
o VLDL 3) HBA1C is requested every after 3 months for
4) What is the test that represents end product of checking and part of TDM for diabetic patient. What
purine metabolism in humans? Uric Acid is the preferred anticoagulant for this test? EDTA
• **“purine metabolism” → Uric Acid 4) SGOT, SGPT, Bilirubin are test for what organ
• **↑ BUA → Gouty Arthritis function? Liver

FEIR, K. ♡
5) Creatinine is the test represents muscle • **Liver is composed of hepatocyte cells – to make
metabolism and is usually a part of what organ it safe/intact → Kupffer cells – macrophage
function test? Kidney monocyte sa blood circulation.
6) What is the anticoagulant needed to examine 21) What is the laboratory phase that LIS (Laboratory
arterial blood gas? Heparin Information System) is needed to continue the
• **EDTA – CBC, HBA1C process? Post-analytical
• **Gray – FBS 22) What is the end product of purine metabolism?
7) In Quality Control, bilirubin test should be analyzed Uric Acid
in an amber vial because it is sensitive to ____ 23) What is the storage temperature if the request is
exposure. Light SAVE SERUM? Freezing Temperature
• **Exposure to light will ↓ result • **Save Serum – Freezer (Storage)
8) What is the characteristic of the serum sample that • Room Temp. → pwede i-test for 8-12 hours
must be avoided in most Clinical Chemistry • Ref → up to 72 hours
procedures? Hemolyzed • 30 days pwede isave ang serum → kapag mahirap
9) What is the serum characteristic when LIPID profile extractan ang patient
is expected to elevate? Chylous or Lipemic 24) 25-30% of blood supply is from the liver, what is the
10) What anticoagulant is considered the naturally function of liver for that process? Vascular
occurring anticoagulant? Heparin • **Blood vessels
11) This is a standard reagent or sample that used to o Artery & Vein → “Vascular System”
set the machine into zero before the running of all 25) What is the test that needs a fasting hours of 12-14
samples. Blank hrs? Triglycerides
• **Blank:
o Distilled water “Water Blank”
o Empty tube “Air Blank”
o Reagent only “Reagent Blank”
o “Serum Blank”
o “Control” – for Quality Control (QC)
▪ Serum and Control ay binibili
mismo, hindi siya kasama sa kit
12) What is the phase in lab. testing that is very
crucial? Pre-analytical
13) How long should the tourniquet be applied in the
arm for venipuncture procedure? 1 minute or less
14) This term implies for a bluish-black discoloration of
the skin due to traumatic puncture. Hematoma
15) The laboratory phase that involves a systematic
error. Analytical
16) The laboratory phase that interpretation of result is
validated. Post-analytical
17) What is the anticoagulant that act as anti-glycotic
agent? Fluoride
18) What is the acceptable Standard Deviation (SD) for
Quality Control Management? ± 2SD
19) Laboratory needs to maintain a Standard Deviation
(SD) in running how many controls before testing
the sample? 3
• **3 Control → Low, Normal, High
20) The Kupffer cells is the microscopic structure of the
liver responsible for? Phagocytosis

FEIR, K. ♡
MT301B – Clinical Chemistry 2 (Lecture)

January 25, 2023 – Introduction & Recall (cont.)

LIVER
• Largest organ
• Weight: 1500–1600 g (1.5–1.6 kg)
• Reddish brown triangular pyramid shaped in rt.
hypochondrium and most of epigastrium.
o **X reddish brown, and it is yellowish brown
→ Fatty Liver
• Lobes: Left and Right → divided by falciform
ligament
o **Mas malaki ang right lobe dahil may 2
minor lobes pa siya:
▪ Quadrate
▪ Caudate
• Functionally divided into lobes by portal vein into 8
lobes.
o Each lobe has: portal vein, branch of
hepatic artery, and bile canaliculi.

• The blood leaves the sinusoids via a central vein,


BLOOD SUPPLY which drains in the hepatic vein.
• Liver receives 1500 mL of blood per minute • Claude Couinad
(blood/min) o French surgeon and anatomist who made
• Blood supply reservoir → can supply and produce significant contribution in the field of
• **Mas marami ang supply from portal vein (80%) hepatobiliary surgery.
kesa hepatic artery (20%) dahil mas malaki ang o First to describe segmental anatomy of the
vein. liver.
• Terminal branches of the hepatic portal vein and
hepatic artery empty together and mix as they enter
sinusoids in the liver. FUNCTIONS OF LIVER
o Sinusoids are distensible vascular
channels lined with highly fenestrated 1. VASCULAR FUNCTIONS
endothelial cells and bounded • Normal blood flow: 1500 ml/min
circumferentially by hepatocytes. • 25–30% from Hepatic Artery
• 70–75% from Portal Veins

FEIR, K. ♡
• Hepatic artery supplies 45–50% of liver’s oxygen A. CARBOHYDRATE METABOLISM
requirement • The final products of carbohydrate metabolism are
• Portal veins supplies the remaining 50–55% glucose, fructose, and galactose.
• The total blood flow from this dual supply • With exception of large amount of fructose that is
represents 25–30% or cardiac output. converted by liver to lactate, hepatic conversion of
fructose and galactose into glucose makes glucose
RESERVOIR FUNCTION
metabolism final common pathway for most
• Portal vein pressure only about 7–10mmHg but the carbohydrates.
low resistance of hepatic sinusoids allows rel. large • All cells utilize glucose to produce energy in form
blood flow through portal vein. of ATP via glycolysis or citric acid cycle.
• Small changes in hepatic venous tone thus can • Liver can also utilize the phosphogluconate
result in large changes in hepatic blood volume, pathway which not only provides energy but also
allowing liver to act as a blood reservoir. produces an important cofactor in the synthesis of
fatty acids.
• Most of glucose absorbed from meal is stored as
glycogen in liver.
• When glycogen storage is exceeded in liver,
glucose is stored as fat.
• Only liver and muscle can store significant amounts
of glycogen,
• Liver and kidney are unique in their capacity to form
lactate, pyruvate, amino acids, and glycerol.
• Hepatic gluconeogenesis is vital in the
maintenance of a normal blood glucose
concentration.
• Glucocorticoids, catecholamines, glucagon &
thyroid hormone greatly enhance gluconeogenesis
BLOOD CLEANSING FUNCTION – whereas insulin inhibits it.
• Kupffer cells lining sinusoids are part of monocyte
– macrophage system. B. FAT METABOLISM
• Functions: phagocytosis, processing Ag, release of • When carbohydrate stores are saturated liver
various proteins, enzymes, cytokines, and other converts the excess ingested carbohydrates into
chemical mediators. fat.
• Phagocytic activity is responsible for removing • Fatty acids thus formed can be used immediately
colonic bacteria and endotoxin entering and stored in adipose tissue or the liver for later
bloodstream from portal circulation. consumption.
• Cellular debris, viruses, proteins and particulate • Only RBCs and renal medulla can utilize only
matter in blood are phagocytosed. glucose.
• Neurons normally utilize only glucose but after a
2. METABOLIC FUNCTIONS few days of starvation they can switch to
breakdown products of fatty acids that have been
1. Carbohydrate Metabolism
made by liver as an energy source.
2. Protein Metabolism
3. Fat Metabolism
4. Drug Metabolism C. PROTEIN METABOLISM
5. Other Metabolic Functions • Liver performs an important function in protein
metabolism
• Steps:
1. Deamination from amino acids

FEIR, K. ♡
o Necessary for conversion of excess amino ▪ Transport proteins (Transferrin,
acids to carbohydrates & fats Haptoglobin, Ceruloplasmin)
o Enzymatic processed convert amino acids ▪ Complement
to their respective keto acids & produce ▪ α1–acid glycoprotein
ammonia. ▪ C – reactive Protein (CRP)
o Deamination of alanine plays an important ▪ Serum Amyloid – A
role in hepatic gluconeogenesis.
o Liver normally deaminates most of amino D. DRUG METABOLISM “XENEBIOTIC
acids derived from dietary proteins METABOLISM”
o Branched chain amino acids are primarily
• Xenebiotic Biotransformation is divided into:
metabolized by skeletal muscle.
o Phase 1 reaction
o Phase 2 reaction
2. Formation of urea
o Phase 3 reaction
o Ammonia formed from deamination is
• Factors affecting drug biotransformation:
highly toxic to tissue
o Genetic factors
o 2 molecules of ammonia + CO2 – Urea
o Diet
o Urea thus formed readily diffuses out of
o Environment
liver and can be excreted by kidneys
o Age
o Enzyme induction/inhibition
3. Interconversion between non-essential
o Liver disease
amino acids
o Cardiac disease
o Hepatic transamination of appropriate keto
acid allows formation of non-essential
amino acids & compensates for any dietary PHASE I
deficiency in these amino acids • It is oxidative hydrolysis & reduction reactions
• It is mainly microsomal oxidases, CYP isozymes
4. Formation of plasma proteins super family.
o Nearly all plasma proteins with notable • These CYP isozymes are concentrated in the
exceptions of Ig are formed by liver. centrilobular zone.
o Quantitatively the most important of these • It needs NADPH for its reactions and hence
proteins are albumin, α1–antitrypsin, and formation of superoxides and reactive free
other proteases/elastases radicals, more chance of injury to these cells,
o Proteins produced by liver: necrosis.
▪ Albumin – maintains normal
plasma oncotic pressure and is PHASE II
principal binding & transport protein • Conjugation with the endogenous hydrophilic
for fatty acids and large no. of molecules.
hormones and drugs. • It involves several processes such as
▪ All coagulation factors which glucuronidation, sulphation, methylation,
exception of factor VIII & von Wille acetylation.
Brand factor are produced in liver. • Glucuronidation is the common type.
▪ Vit. K – necessary cofactor in • Hepatic microsomal uridine diphosphate
synthesis of Prothrombin, factor glucuronyl transferase mediates the reaction.
VII, IX & X
• These are susceptible to enzyme induction.
▪ Liver also produces plasma
• Heavy smoking, phenytoin administration seen
cholinesterase, an enzyme that
to increase glucuronidation in humans.
hydrolyses esters, including Local
• In some drugs the conjugation ends up with a
anesthetics & Sch
metabolite more potent than the parent drug.
o Other important proteins produced are:
o Ex: Morphine becomes morphine 6–
▪ Protease inhibitors (antithrombin
glucuronide a potent byproduct which is
III, α1–antitrypsin)
FEIR, K. ♡
responsible for some of the analgesia E. OTHER METABOLIC FUNCTIONS
produced by morphine. • Liver plays an important role in hormone, vitamin &
mineral metabolism
PHASE III • Normal thyroid function is dependent on hepatic
• It is an energy mediated transport/ elimination by formation of the more active T3 from T4
ATP- binding cassette transport proteins. • Liver is also major site of degradation for insulin,
• Facilitates excretion of xenobiotics and steroid hormones, glucagon & ADH
endogenous compounds. • Hepatocytes are principal storage sites for Vit A,
• These proteins use ATP hydrolysis to drive B12, E, D & K.
molecular transport. • Hepatic production of transferrin & Haptoglobin is
• These resides on the canalicular surfaces of important because proteins are important in iron
hepatocytes and enables biliary excretion of hemostasis.
cationic compounds, including anticancer drugs. • Coagulation
o Hepatocytes make most of the pro-
coagulants with exceptions of Factors III,
IV, VIII.
HEPATIC DRUG CLEARANCE
o Liver also makes protein regulators of
• Rate of hepatic blood flow, protein binding, hepatic
coagulation & the fibrinolytic pathways.
intrinsic clearance. o Such regulators include protein C, S, Z,
• Drug elimination – volume of blood from which the Plasminogen Activator Inhibitor, &
drug is completely removed per unit of time. antithrombin III
• Is equal to the product of hepatic blood flow and the
extraction ratio.
• Extraction ratio(E): amt of drug removed from the 3. BILE FORMATION AND EXCRETION
blood during a simple pass through the liver. • Heme metabolism
• Anesthetics significantly alter extraction by o Main site
reducing hepatic blood flow. o Hemoglobin is heme and globulin, with
• Inhalational agents may influence drug clearance heme containing ferrous and porphyrin IX
by altering drug-metabolizing ability or intrinsic o 20% approx., heme synthesized in the liver
clearance o Rate limiting step is synthesis of 5 –
• Inhalational agents have been shown both in vitro aminolevulinic acid catalyzed by ALA
and in vivo to alter drug metabolism at clinically synthetase
relevant concentrations. • Bilirubin metabolism
• They are competitively inhibiting p-450, and phase o Source is from the heme metabolism
II reactions. o Approx. 300mg of bilirubin formed
everyday
o 80% by the phagocytosis of scenecent
High hepatic Low hepatic extraction RBCs by the RE cells
extraction ratio ratio o The extracted heme is converted to
Lignocaine Diazepam bilirubin → this is the rate limiting step
Propranolol Thiopentone o This is then bound to albumin and liver
Meperidine Theophylline processes the molecules into conjugated
Verapamil Digitoxin bilirubin in 2 steps, and then excreted
Phenytoin o Enterohepatic circulation ensures some of
Pancuronium these products to return to the liver.

FEIR, K. ♡
• Lipid metabolism
o End product: Fatty Acids → excess ay
mapupunta sa adipose tissue (fats that can
be a reserved energy)
• Protein (CHON) metabolism
o End product: Amino Acids
o Kjeldall Method – binding

3. Bile Formation and Excretion


• 120 days – life span of RBC
o Dead RBC → spleen “Graveyard Shift”
o Sa spleen → Hgb degradation → chopped
into heme and globin

Hemoglobin degradation

REVIEW OF ANATOMY & PHYSIOLOGY:


FUNCTIONS OF THE LIVER
• Carbohydrate metabolism
o Glycogenesis
o Glycogenolysis
o Gluconeogenesis
• Fat metabolism
o Ketogenesis
• Protein Metabolism
o Anabolism
o Deamination
o Urea formation
• Secretion of bile
• Detoxification
• Metabolism of Vit. A, D, K, E
• Clotting factors (Prothrombin)
• Storage
• Blood store

ADDITIONAL NOTES:
1. Vascular Functions “Blood Reservoir”
• Supply blood sa veins, arteries
• EMERGENCY: Internal hemorrhage → will release
300 mL of blood dahil hindi na kakayanin ni bone
marrow

2. Metabolic Functions
• Carbohydrate (CHO)
o Storage form: Liver
o Magiging glycogen

FEIR, K. ♡
MT301B – Clinical Chemistry 2 (Lecture) CARBOHYDRATES
February 1, 2023 – Lipid Profile Test (cont.) and
PRE-TEST
Carbohydrates
1. Which of the following given is an example of
simple compounds of CHO in the body that gives
energy to cells? Glucose
LIPID PROFILE TEST 2. Which is the non-reducing form of CHO? Sucrose
1. Bilirubin – TB, B1, and B2 • **Carbohydrate came from “Hydrates of Carbon” →
• Screening test: detection of problem in liver has H2O and combination of Carbon
2. Total Protein Albumin Globulin Ratio (TP/AG) • Sucrose is water soluble
3. Liver Enzymes – Alanine Transaminase (ALT) 3. Which among the choices are disaccharides?
and Alkaline Phosphatase (ALP) Maltose, Lactose, Sucrose
• ALT partners with AST • **Most methods starts with complex process ->
4. Virology – Viruses Formation of polysaccharide and ang end product
• Hepa A, B, C, D, E, G na itetest mismo ay ang monosaccharide
(simplest)
4. What is the storage form of carbohydrates in the
METABOLISM OF LIVER body? Glycogen
1. Carbohydrate (CHO) Metabolism • Monosaccharide “Hexoses” – has 6 Carbons
• Storage: Glycogen o Glucose – form of sugar that is readily
• End product (waste CHO metabolism): Carbon converted to energy (dextrose)
dioxide and Water o Fructose – hexose sugar found in honey
o Monosaccharides: (GGF) and fruit.
▪ Glucose, Galactose, Fructose o Galactose – sugar of the hexose class
o Disaccharides: (LMS) which is a constituent of lactose and many
▪ Lactose, Maltose, Sucrose polysaccharides.
▪ Sucrose – non-reducing sugar • Di/Oligosaccharides
o Polysaccharides: o Sucrose – fructose + glucose (table sugar)
▪ Glycogen, Starch, Cellulose o Lactose – galactose + glucose (milk sugar)
▪ Cellulose – plant cell wall → gives o Maltose – glucose + glucose
rigidity sa vegetable • Polysaccharides
2. Lipid Metabolism o Most abundant CHO
• End product: Fatty Acid o Glycogen, Starch (bread, root crops),
• Lipids are stored in Adipose Tissue Cellulose
• Lecithin
• Cholesterol Additional Notes
o Lipogenesis – metabolism of liver → • **Fruit diet:
carbohydrate source (ice cream, sweets) o Apple & Banana → Good for breakfast and
will convert into lipids/fats lunch
3. Protein (CHO) Metabolism o Oatmeal → X night because it causes
• Production of RNA and DNA kabag; Polysaccharide
• Transport protein → Albumin & Globulin o Melon & Watermelon → Good for digestion
o “Transport protein” so automatic kailangan before dinner
sila sa circulation 5. What is the metabolic end product of CHO in the
4. Drug Metabolism body? Water and Carbon Dioxide
• Toxic drugs to less toxicity • Carbohydrates, when utilized as fuel produce
o **Inaalis ng liver ang toxin na galing sa carbon dioxide and water as waste products of
gamut “metabolize” metabolism.
o **2 CHON enzymes in metabolism:
▪ Amylase – salivary glands (mixes
while chewing)
FEIR, K. ♡
▪ Lipase – pancreas
(monosaccharide)
▪ Request Form: Amylase (AMS),
Lipase (LPS)
• Carbohydrates can also be used in biological
substances such as glycolipids, glycoproteins,
heparin, nucleic acids, and other substances.
• When our body takes in carbohydrates with our
diet, the ingested disaccharides (Sucrose and
Lactose), as well as polysaccharides (Starch and
Glycogen) are converted to monosaccharides by
the action of amylase.
6. Rich food in CHO when taken into the mouth to
enter into the body, is not digested to what part of
the digestive system? Stomach • Insulin – hypoglycemic agent (lowers the glucose
• **Initial digestion of CHO: Mouth → Stomach → level in the blood)
Small Intestine o **Maraming kinain = ↑ Insulin (after 30
o In stomach, nagppass through lang and mins) to metabolize sugar. Then, papasok
paikot-ikot. si glucagon if ↓ na si insulin.
▪ 2 Digestion: Proteins and Lipids • Produced by the beta cells of the pancreas
o In small intestine, mix with pancreas na (Diabetes Mellitus Type I and II)
magddigest/form into monosaccharides.
o Excess: Liver Type I Type II
• **Know if ↓ glucose level in brain: Nahihilo and Insulin Dependent Non-insulin Dependent
nanginginig ang muscle (IDDM) (NIDDM)
o It means, wala nang reserve energy < Age > 40 age
(glycogen). With this, tatataas ang Genetically Diet, Acquired
hormone glucagon with the help of Normal weight Obese
pancreas. X insulin Insulin receptor
o Glucagon – “Glucose is Gone” **Insulin receptor – nagpproduce kaya lang hindi
• Final absorption: Small Intestine → Liver tinatanggap ng cell para mametabolize
(disaccharide enzyme – monosaccharide)
• Digestion (stomach) no CHO • Glucagon – hyperglycemic agent (elevates blood
• Enzymes that are able to split disaccharides are glucose level) alpha cells of the pancreas (“glucose
known as glycosidases or disaccharides. is gone” lower na ang blood sugar)
7. What is the metabolism when there is a conversion • Lipogenesis – formation of fat from carbohydrates
of glucose to glycogen? Glycogenesis (Ice cream, cake)
• Breakdown → “glysis” • Fats → fatty acids (liver) – stored (adipose tissue)
• Formation → “genesis” for the signal to be released. Reserve stored
• Non → “neo” energy.
8. What is the carbohydrates metabolism when there • Ketogenic diet. Fatty meal – Glucose
is a formation of glucose from non-carbohydrate • Liver (Fatty liver) – Cirrhosis
sources? Gluconeogenesis • Fat metabolism – Ketone bodies (urine)
9. When glucose and other hexoses converts into 10. In a specimen collected for plasma glucose
pyruvate or lactate, what is the metabolism action? analysis, sodium fluoride___. Inhibits glycolysis
Glycolysis
• **X Glycogenolysis kasi: Glycogen → Glucose
(Storage form complex na kailangan ibreakown
kaya may “lysis”)

FEIR, K. ♡
SPECIMEN COLLECTION ▪ Ferric Reduction
• Specimen of Choice (SOC) ▪ Condensation
o Whole blood – EDTA (CC – HgbA1C) o Enzymatic:
o Serum – liquid portion of the clotted ▪ Glucose Oxidase
o Plasma – unclotted blood ▪ Hexokinase G6PD
o CSF – gray tube 12. Which of the following would be an example of
glucose-specific-colorimetric method?
Tube Test Storage a) Alkaline ferricyanide
Tube 1 Clinical Chemistry Freezing temperature
b) Glucose oxidase
Tube 2 Bacteriology Room temperature for 30
c) Hexokinase
mins, 35-37 C
Tube 3 Hematology Refrigeration d) O-toluidine
temperature 13. Increased concentrations of ascorbic acid inhibit
Tube 4 Serology chromogen production in which of the following
If 1 vial only: Bacteriology → Clinical Chemistry → glucose methods?
Hematology a. Ferricyanide
o Serous Fluids b. Ortho-toluidine
▪ Exudates (↑ value than c. Glucose oxidase (peroxidase)
transudates) d. Hexokinase
▪ Transudates (Clear) 14. The most specific method for the assay of glucose
o Urine – creatinine clearance (24 hours) utilizes:
o Synovial Fluid – joints a. Hexokinase
b. Glucose oxidase
• Standard SOC – fasting venous plasma c. Glucose-6-phosphatase
o Fasting: 6-8-10 hours (overnight) d. Glucose dehydrogenase
o Venous: has lower glucose than arterial • Glucose oxidase is not the reference method:
blood. It is easier to collect o It is too specific
o Anticoagulant: FBS (Gray-Fluoride) ▪ Reacts with beta-glucose (65%)
o Plasma: whole blood gives approximately and alpha-glucose (35%)
10-15% lower glucose levels than serum or o It is affected by interferences
plasma. ▪ Oxidizing (Soap)
11. In fasting state, the arterial and capillary blood ▪ Reducing agents (Vit. C/Uric acid)
glucose concentration varies from the venous • Hexokinase G6PD is the reference method for
glucose methods. glucose analysis
a. 2 mg/dL higher o Hexoses + ATP → Hexose 6 Phosphate +
b. 5 mg/dL higher ADP
c. 10 mg/dL lower o G6P + NAD → 6 phosphogluconate +
d. 12 mg/dL lower NADH (340 nm)
15. In the hexokinase method for glucose
determination, the actual end product measured is
the: NADH produced from the reduction of NAD

• Methods in measurement of Glucose:


o Chemical:
▪ Copper Reduction
FEIR, K. ♡
STANDARD DEVIATION
Example Problem:
76, 84, 69, 92, 58
89, 73, 97, 85, 77
Steps:

1. Get the Mean


76 + 84 + 69 + 92 + 58 + 89 + 73 + 97 + 85 + 77
10
= 𝟖𝟎

2. Get the Standard Deviation

1 76 – 80 (-4)2 16
**Metabolic Disorder – may genetically infect 2 84 – 80 (4)2 16
3 69 – 80 (-11)2 121
**Metabolism – pathway na may sira 4 92 – 80 (12)2 144
5 58 – 80 (-22)2 484
• In man, the frequency of taking meals and the
6 89 – 80 (9)2 81
amount of carbohydrates converted to fat could 7 73 – 80 (-7)2 49
definitely have a bearing in diseases such as: 8 97 – 80 (17)2 289
o Atherosclerosis 9 85 – 80 (5)2 25
o Obesity 10 77 – 80 (-3)2 9
o Diabetes Mellitus (DM) n = 10 ∑ 1, 234
Glucose Profile Test
FBS Fasting Blood Sugar

RBS Random Blood Sugar √(𝑥 − 𝑥)^2
2hrs PPBS Post Prandial Blood Sugar ∑
𝑛−1
OGCT Oral Glucose Challenge Test
OGTT Oral Glucose Tolerance Test √1, 234

HBa1c Glycosylated Hemoglobin 10 − 1
= 11.709 or 11.7 or 12

• NOTE: If the procedure you are performing is only


reported out in whole number, use 12 as the SD.
However, if the procedure results are in decimals,
use the SD in decimals (11.7).

3. To determine ± 2 SD, simply multiply the SD


received in the above method of calculation by 2,
± 3 SD multiply by 3.

± 1 SD
• Mean ± 1 (SD)
o 80 + 1 (11.7) = 91.7 → +1 SD (HIGHEST LIMIT)
o 80 – 1 (11.7) = 68.3 → -1 SD (LOWEST LIMIT)
± 2 SD
• Mean ± 2 (SD)
o 80 + 2 (11.7) = 103.4 → +2 SD (HIGHEST LIMIT)
o 80 – 2 (11.7) = 56.6 → -2 SD (LOWEST LIMIT)

FEIR, K. ♡
± 3 SD Shift
• Mean ± 3 (SD) • When six or more consecutive daily values
o 80 + 3 (11.7) = 115.1 → +3 SD (HIGHEST LIMIT) distribute themselves on one side of the mean but
o 80 – 3 (11.7) = 44.9 → -3 SD (LOWEST LIMIT) maintain a constant level.

4. Graph

16. Term used to describe reproducibility is: Precision


17. The extent to which measurements agree with the
true value of the quantity being measured is known
as. Accuracy
18. The most frequent value in a collection of data is
statistically known as the: Mode
19. Middle value of a data set is statistically known as
the? Median
20. An index of precision is statistically known as the:
Standard Deviation
21. The statistical term for the average value is the:
• **Important na hindi lumampas sa ± 3 ang values, Mean
lalong-lalo na sa ± 2
• **Check if saan ba pumasok ang mga point sa
graph: Glucose control is 100 mg/dL
o Example: Day 8 (97), lumampas sa +2SD. • Plus or minus (±) 5 mg/dL, in order to have
In Westgard rule = 12S accuracy, the results must fall within the limits of
• Westgard Rule: 12S = may lumampas na isa sa 2 100 mg/dL (±)5 mg/dL → Normal is considered
SD as either tumaas ng lima or bumaba ng lima.
o Random Error – Odd number (13s, R4s) o Example: 100, 105, 92, 100, 99, 105, 98
o Systematic Error – Even number (22s,
41s, 10x) • 100 → Normal
• 105 → Normal (105 – 5 = 100 → within N 100)
• 92 → Error (100 - 5 = 95)
Trend • 100 → Normal
• When the value of the control continues to increase • 99 → Normal
or decrease over a period of five or six consecutive • 105 → Normal
days. • 98 → Normal
= 1 ERROR

PRECISION
“REPRODUCIBILITY”
• Ability to produce a series of results that agree
closely to each other.
• Has the closeness of results to each other from the
same unknown.
FEIR, K. ♡
• Example: 90, 91, 89, 92, 90, 93, 88, 91, 89, 90
(± 5)
o Results are very close to each other, is
considered good precision.
o How reliable are your results?

• 91 – 90 = 1
• 89 – 91 = 2
• 92 – 89 = 3
• 90 – 92 = 2
• 93 – 90 = 3
• 88 – 93 = 5
• 91 – 88 = 3
• 89 – 91 = 2
• 90 – 89 = 1
= PRECISE BUT NOT ACCURATE
**Not accurate dahil hindi umabot ang mga data sa
110 mg/dL.

FEIR, K. ♡
MT301B – Clinical Chemistry 2 (Lecture) • **From small intestine, ma-absorb ang urine then
iccirculate in blood. Then, it will be secreted to
March 6, 2023 (Async.) – Liver Function & Enzymes
kidney kaya may urobilinogen.
• **Since may obstruction, si B2 ay nalilimitahan na
pumunta sa small intestine para ma-absorb at
MLS ACTIVITY mapunta sa circulation para lumabas kay kidney
1. Which of the following statements regarding the
metabolism of bilirubin is true? It is produced from
the destruction of RBCs
• **Bilirubin – degradation product of hemoglobin.
Galing sa 120 days old RBC, dadalhin sa spleen,
then spleen will reduce to heme (magtutuloy
papunta kay bilirubin) and globin
2. Which is true of pre-hepatic bilirubin?
Unconjugated
• **Unconjugated is water insoluble and indirect
bilirubin
• **Pre-hepatic – “before” the liver, siya ay cinarry ng 7. Hemolytic jaundice produces an increase in what
albumin. level of bilirubin? Total and Indirect
• **From the spleen, it will be carried out to blood 8. In patient with an elevated indirect bilirubin, what is
stream papunta kay liver. the bilirubin level that is also measured? Total
3. Direct bilirubin is greatly elevated in what clinical bilirubin
condition? Obstructive jaundice 9. Bilirubin is transported from reticuloendothelial
a. Hepatitis → Hepatic; sa liver mismo cells to the liver by: Albumin (protein transporter)
b. Hemolytic anemia → Pre-hepatic; nagllyse na 10. In the liver, bilirubin is conjugated by addition of:
ang RBC Glucoronyl groups
c. Obstructive jaundice → Post-hepatic; need to • **B1 is converted to B2 pag nasa loob ng liver by
consider kung bakit nagtataas si B2, may Glucoronyl groups
humaharang sakanya papunta kay small 11. Which conditions is caused by deficient secretion
intestine (may bara – pwedeng may problem sa of bilirubin into the bile canaliculi? Dubin-Johnson
gallbladder and pancreas) syndrome
• **Direct bilirubin – B2 – Post-hepatic a. Dubin-Johnson syndrome – kulang si B2
• **Indirect bilirubin – B1 – Pre-hepatic papunta kay bile kaya na-stay lang siya sa loob
4. Conjugated bilirubin is also known as: Direct ng liver and will cause dark liver.
bilirubin b. Gilbert disease – pre-hepatic disease (B1
5. In a patient having a hemolytic jaundice condition, cannot enter into liver – it is outside)
what is the expected conjugated bilirubin level? c. Neonatal hyperbilirubinemia – elevated level
Normal or slightly elevated of B2
• **Hemolytic jaundice – pre-hepatic d. Crigler-Najjar syndrome – liver converted B1
can be less or no; absence of enzyme
• **B1 → Liver (papadaanin lang dito) → B2
o If normal si liver, normal din niya irrelease
si B2 kahit super dami ng B1. But, if super Indirect Bilirubin Direct Bilirubin
dami ni B1 and liver is napapsukan ng • B1 • B2
super daming B1, then need din niya mag- • Pre–hepatic → • Post–hepatic →
release ng maraming B2 Hemolytic Anemia Obstructive
6. Urine urobilinogen is below normal or negative in
• Conjugated Jaundice
patient with what clinical condition? Obstructive
• Unconjugated
jaundice

FEIR, K. ♡
JAUNDICE • Abnormal excretion (hindi
naeexcrete)
• Harmless, no RX

12. Which form of hyperbilirubinemia is caused by an


inherited absence of UDP-glucuronyl transferase?
Crigler-Najjar syndrome
13. Which form of jaundice occurs within days of
delivery and usually lasts 1-3 weeks, but is not due
to normal neonatal hyperbilirubinemia or hemolytic
disease of newborn? Lucey-Driscoll syndrome
• **Mga pinapanganak na naninilaw; kaya need ma-
check ang baby ng bilirubin level

CONGENITAL HYPERBILIRUBINEMIAS LUCEY–DRISCOLL SYNDROME


• Due to abnormal uptake / abnormal conjugation / • Rare form of jaundice caused by unconjugated
abnormal excretion of bilirubin. bilirubin that presents within 2-4 days of birth and
can last several weeks.
• AR (Autosomal Recessive) • Caused by inhibitor of UDP-glucuronyl transferase
• Type 1: total absence of UDP in maternal plasma that crosses the placenta.
glucuronyl transf. → no
conversion of bilirubin (B1 →
B2)
Crigler-Najjar • Type II: partial def. of UDP GT
Syndrome • Unconjugated Bilirubin
>20mg/dL (**since hindi
nacconvert si B1 to B2)
• Kernicterus (**high level of
Bilirubin in brain)
• Death: 1 yr of life
• AD inheritance (Autosomal
Dominant)
• Males
Gilbert’s • Defective uptake of bilirubin by
Syndrome the liver (**hindi napapasok sa
PRE-HEPATIC
liver)
• Unconjugated Bilirubin 3mg/dL • Hemolytic Anemia
• Serum
• Harmless, no treatment (Rx)
o B2 – normal or slightly elevated
• AR o B1 – elevated
• Defective excretion of conj. • Urine
bilirubin into bile (**converted o Urobilinogen – very elevated
into B2 into the liver but cannot o Bilirubin – negative or normal
Dubin-Johnson
excrete in liver kaya nag- • Stool
Syndrome
accumulate ang bile) o Urobilinogen – very elevated
• Mutation in gene encoding
MOAT protein HEPATIC
• Black liver jaundice • Hepatitis, viral or drug induced
Rotor • AR • Serum
Syndrome • Exact cause?? o B2 – slightly elevated
o B1 – greatly elevated

FEIR, K. ♡
o **If may problem si liver, mahihirapan BILIRUBIN
siya to convert B2. So mas • Clinical significance:
dumadami/naiipon si B1 sa labas or o Hepatitis
circulation. o Cirrhosis
• Urine o Other liver cirrhosis
o Urobilinogen – elevated (**since o Biliary obstruction (gallstones, CA)
maraming B2 ang lumalabas na • Urine Bilirubin & Urobilinogen in Jaundice
naabsorb ni intestine papunta sa blood
stream to be excreted by kidney) UREA BILIRUBIN & UROBILINOGEN IN
o Bilirubin – present JAUNDICE
• Stool
o Urobilinogen – decrease or negative Urine
Urine Bilirubin
Urobilinogen
o **Opposite – if urobilinogen is elevated
Obstructive
kay urine, nababawasan ang pag-punta +++ Normal
jaundice
niya kay large intestine kaya Liver damage + or - ++
nagddecrease or negative ang Hemolytic
urobilinogen sa stool. Neg +++
jaundice

POST-HEPATIC
15. Hyperalbuminemia is caused by: Dehydration
• Extra/obstructive jaundice
• Serum syndromes
o B2 – greatly elevated (**since may • **Total Protein Albumin Globulin Ratio (TPAG) –
obstruction, bumabalik sa circulation. test for liver function. Once there is albumin
Hindi masyado makapunta kay intestine, increase → dehydrated ang patient
kaya naman dadalhin ni liver sa portal • A/G ratio
circulation or blood)
o Normal ratio is albumin > globulin.
o B1 – only slightly elevated
o Reported in grams/dL
• Urine o Any disturbances will lead into:
o Urobilinogen – normal to below normal or ▪ Albumin < Globulin → Nephrosis,
negative (**may obstruction, hindi siya multiple myeloma (cancer of
makapunta kay kidney) plasma cells)
o Bilirubin – elevated (**bilirubin pupunta
ulit sa circulation in the form of B2 which
is water soluble) LIVER AND ANAESTHESIA
• Stool • Anesthesia & anaesthetic drugs affects the hepatic
o Urobilinogen – below normal or negative function by following mechanisms:
o **Gray stool – less ang pag-convert into
o Alteration in the hepatic blood flow n HABR
urobilinogen into urobilin para
magkaroon ng color ang urine. o Metabolic function
o Drug metabolism
o Billiary function

14. What is the color of the feces if patient was


diagnosed of an obstructive jaundice? Gray HALOTHANE HEPATITIS
• **Dark brown – hepatic diseases (Hepatitis) • It is immunologically mediated, as it induces both
• **Tarry (black) – parasitism, abnormal bleeding in neoantigens & auto antigens. The incidence of
GI tract (upper;stomach) fulminant hepatic necrosis terminating in death
associated with halothane was found to be 1 per
35,000.
• Demographic factors: It’s an idiosyncratic reaction,
susceptible population include Mexican
Americans, Obese women, Age >50 yrs, Familial

FEIR, K. ♡
predisposition, Severe hepatic dysfunction while
Children are resistant.
• Prior exposure to halothane is a important risk
factor & multiple exposure increases the chance of
hepatitis.

TESTING FOR SP. DISEASES


• Targeted testing is used to identify specific hepatic
or biliary diseases.
• Examples:
o Serologic testing – identify viral, microbial
& autoimmune causes.
o Genetic testing – diagnose heritable
metabolic disorders.
o Tumor marker assays – detect hepatic
malignancies.
• Identifying viral markers (antibodies, antigens and
genetic material) – key for diagnosis of hepatitis
from hepatotropic viruses (A, B, C, E) and
herpesviruses such as CMV & EBV.
• Special tests – serum α1 – AT and phenotype
analysis. Markers for hepatic malignancy – AFP,
des – γ – carboxylated prothrombin.

OTHER TESTS
• CBC – Hb may show anemia esp with the target
cells in jaundiced patients due to macrocytosis.
• Leucopenia – complicates portal HTN and
hypersplenism.
• Leucocytosis – in hepatic abscess, alcoholic
hepatitis, cholangitis.

Thrombocytopenia – in cirrhosis, due to dec in


thrombopoietin in liver, and hypersplenism.

FEIR, K. ♡
MT301B – Clinical Chemistry 2 (Lecture) SPECIFICITY
• Active sites
March 6, 2023 (Async.) – Liver Function & Enzymes
• Specific interaction of the enzyme that promotes
(cont.)
the formation of the transition state.

ENZYMES
• Biomolecules that catalyzes biochemical pathways
• Speeds up chemical reaction without themselves
undergoing any permanent chemical change.
• They are neither used up in the reaction nor they
appear as reaction products. (**hindi sila
nagagamit, only means of transporting and to
convert product)
• **Once combined → ES Complex
• **Kidney bean shaped
• Weak interactions at the active site: (**hindi pa
ganon ka-fix ang substrate attachment)
o Electrostatic interactions (marami si +,
magkakaroon ng less interaction; if – naman,
walang formation) **+ and -
o Hydrogen bonds
• Lowers the activation energy
▪ Ionic – transferring
o Activation energy or energy of activation is the
▪ Covalent – sharing
amount of energy required for a reaction to
o Van der Waals forces
proceed.
▪ > Forces of attraction = ↓ Distance
▪ < Forces of attraction = ↑ distance
o Hydrophobic interaction

ENZYME-SUBSTRATE (ES) FORMATION


• How they are formed:
o Lock and Key Model
o Induce-fit Model

LOCK AND KEY MODEL


STRUCTURE • Rigid, not movable, cannot be manipulated.
• Globular protein
• Made up of long, linear chains of 62–2,500 amino
acids that fold to produce a three dimensional
conformation
• Each unique amino acid sequence produces a
specific structure, which has unique properties.

INDUCED-FIT MODEL
• Flexible
• There is manipulation of conformation at the active
site when the substrate is to bind it.
FEIR, K. ♡
6 Ligases Ligation of 2 substrates at
the expense of ATP

FACTORS THAT AFFECT THE FUNCTION OF


ENZYMES

TEMPERATURE
• Rate of enzyme activity increases with an increase
temperature. However, at some point temperature
rises denature protein and enzyme activity decline
sharply.

CONCENTRATION
• Increase in concentration of substrate or enzyme
increases the reaction rate and vice versa.

pH
• Enzymes work best at its most favorable pH value
– the point where the enzyme is most activate is
known as the optimum pH.
CLASSIFICATION
(based on reactions they catalyze)
Class Type of reaction
1 Oxidoreductases Oxidation-reduction
2 Transferases Group transfer
3 Hydrolases Hydrolysis reaction
4 Lyases Addition or removal of
groups to form double
bonds
5 Isomerases Isomerization

FEIR, K. ♡
INHIBITORS TYPES OF ENZYMES
• Substances which alter the catalytic action of the • Apoenzyme (incomplete enzyme)
enzyme and consequently slow down, or in some o Protein portion of the enzyme
cases, stop. • Holoenzyme (complete enzyme)
o apoenzyme + co-factors (non-protein)
o Co-factors:
▪ Co-enzyme (organic)
▪ Activators (metallic)
▪ (non-metallic)
• Zymogens or Proenzymes
o Inactive form of enzymes
o **Need ng combination to form other
• Macroenzymes
• **Competitive – siya mismo nag-aattach sa active o Big in appearance
site o Conjugated to an immunoglobulin or any
• **Non-competitive “Allosteric Site” – sa labas large molecule
ng enzyme siya nag-attach • Isoenzymes
o Catalyze the same reaction but differ in
A. Competitive Inhibition terms of physical or chemical
• Occurs when the substrate and a substance characteristics and tissue distribution
resembling the substrate are both added to the o Example: ALP (4), CK (3), LDH (5)
enzyme. o **Iso means isa lang → Same action but
different location or it can be found in other
cell distributions.

NOTE:
• Enzymes are measured according to activity not
by concentration.
1. International Unit (IU) – measured
micromole of substrate per minute
**’pag magreread ng enzyme after
B. Non-competitive Inhibition incubation of 1 minute, dun pa lang ang initial
• Occurs when substances which when added to the reading. Another minute for final reading.
enzyme alter the enzyme in a way that it cannot 2. Katal Unit (KU) – measured mole of
accept the substrate substrate per second.
**30 seconds lang ‘pag first reading, then
another incubation period of less than 60
seconds to 8 minutes.

FEIR, K. ♡
MT301B – Clinical Chemistry 2 (Lecture) Obstructive Albumin is slightly decrease
jaundice and gamma globulin is
March 6, 2023 (Async.) – Liver Function & Enzymes
slightly increased
(cont.)
Chronic Decreased albumin and
hepatitis/Cirrhosis increased globulins
TEST FOR LIVER FUNCTION

A. PROTEINS B. LIVER ENZYMES


• Method of Protein Analysis: • **
o Refractive Index C. AMMONIA
o Biuret Method – most commonly used • **Test to determine toxification function of liver
method • Specimen considerations:
o Turbidimetric o Serum is NOT used; EDTA plasma or
o Electrophoresis heparinized plasma can be used.
• Principle: Whenever an alkaline copper reagent o Placed immediately on ice bath
acts with substances containing two or more o Analyzed immediately (20 minutes)
peptide bonds, it produces a blue violet color o No to hemolysis
• Clinical significance: Reye’s syndrome
TP A/G
(correlated with CSF – Glutamine test)
• Total Protein – Biuret Method
• Albumin – Dye Binding Method
o BCG – binds to albumin producing a color
that is read spectrophotometrically
o BCP – may also be used

D. BILIRUBIN

CLINICAL CASE

Chronic liver Decrease in albumin and


cirrhosis increase in gamma globulin,
beta globulin
**Can develop into liver
cancer
**The normal is albumin is
greater than globulin. But
once may decrease sa
albumin, and increase in
globulin, there is a disease
affecting liver.

FEIR, K. ♡
A. ALKALINE PHOSPHATASE (ALP)
• **Sa enzymes na ito. Enzyme wherein the pH is
controlled at alkaline stage
• The normal serum of ALP is the result of two
factors:
o The rate of release of ALP from tissues,
mainly liver and bone.
o The rate of inactivation and excretion
(depending on the activity)
• The liver is the main source of serum ALP
• In liver disease, the increased level comes from
increased release from the liver cell and also from
the regurgitation or “backing up” of the biliary
isoenzymes back into the serum. **if may problem
sa liver, abnormal ang formation → magkakaroon
ng back and forth or upper and lower ng ALP
• ALP is much more greatly elevated in
obstructive jaundice than in liver disease.

HEPATITIS
• Since there is damage to the liver cells in liver
disease, the enzyme in the cells are released,
giving elevated ALP but since this enzyme is
produced by the liver cells, the production
diminishes, thus keeping the levels from being
extremely elevated.
o **Kaya pag may problem ang liver,
nagnnormalize pa rin ang ALP. Pero
kapag hindi liver ang may problem,
bumalik lang sakanya ang ALP or
enzymes because hindi siya naka-push
through sa intestine kaya nag-eelevate.
• The liver cells being diseased cannot synthesize
adequate amounts of ALP, consequently blood
levels do not rise much above normal.

OBSTRUCTIVE JAUNDICE
• The enzyme that is secreted by healthy liver cells
into the bile, backs up or is regurgitated back to
the liver, thus giving higher levels of enzyme than
normal.
• As the liver produces more and excretes more
enzyme all the time, because of the obstruction,
the regurgitated or backed up enzyme builds up
higher and higher in the bloodstream giving very
high ALP levels.

FEIR, K. ♡
• Alkaline Phosphatase Isoenzymes: • Pregnant and growing children
o Placenta
o Intestine
o Liver
o Bone \
• Identified and separated by:
1. Electrophoresis (I-P-B-L) – fastest to migrate
is the liver. 4321

2. Heat denaturation – serum is heated 56C for


4321
10 minutes. (P-I-L-B).
o Most heat stable is Placenta
o Most heat labile is Bone

• **Positive yung may dark color. Negative ang light


color
• **Ang pag-read ay pataas

3. Chemical Inhibition
o L-phenylalanine – inhibits placental, B. GAMMA GLUTAMYL TRANSFERASE
intestine, Regan and Nagao (GGT)
isoenzymes • Used for evaluation of hepatobiliary disease
o Levamisole - inhibits liver and bone • Elevated levels may indicate alcoholism
isoenzymes • Can be used as a means to differentiate the cause
o Urea – inhibits bone isoenzyme of elevation of ALP
o Leucine – inhibits Nagao isoenzyme • GGT – transfers a y-glutamyl residue to an amino
acid (transpeptidation)
CLINICAL SIGNIFICANCE
• This function is involved in peptide and protein
• Often used in evaluated of hepatobiliary synthesis
(obstructive jaundice) and bone disorders • Distribution: Tissue sources of GGT include
• Highest elevation of ALP is seen in Paget’s kidney, brain, prostate, pancreas, and liver. Urine
disease contains significant amounts of GGT.

PHYSIOLOGIC ELEVATION
FEIR, K. ♡
• Measurement: Total activity is by the SZASZ • Late marker for Acute Myocardial Infarction (AMI):
assay. In this reaction, the substrate is y-glutamyl- o Rises: 12–24 hrs
p-nitroanilide, with the release of p-nitroaniline. o Peaks: 48–72 hrs
o Return to normal: 10 days
CLINICAL SIGNIFICANCE
• Catalyzes: Lactate to pyruvate 5 isoenzymes
• GGT levels are elevated in almost all
• Electrophoresis: LD2, 1, 3, 4, 5
hepatobiliary disorders or biliary tract obstruction
• LD flip: LD1>Ld2 (Associated with AMI but not
as well as in patients taking enzyme-inducing
specific)
drugs (Warfarin, Phenobarbital, Dilantin)
• GGT levels are often examined with ALP levels
o If both GGT and ALP levels are high,
some type of liver disorder is
suspected.
o If GGT is normal and ALP is high, bone
disease is likely present.
• Patients with acute pancreatitis exhibit increased
GGT levels. In persons with diabetes, GGT is
typically increased as triglyceride levels rise.
(**Directly proportional)
• Because alcohol has enzyme-inducing
properties, the most useful application for GGT
is the detection of alcoholism and the monitoring
of alcohol intake by patients during treatment. • **Kapag may 6 na → Alcohol dehydrogenase
(great prognosis for death)
• **H – heart; M – muscle

C. 5-NUCLEOTIDASE
• Originates in the liver or diseased bone
• 5’-NT becomes elevated in liver disease as does
ALP but 5’-NT does not increase in patients with
bone disease.
• Serum levels of 5’-NT are more sensitive to liver
cancer.

D. LACTATE DEHYDROGENASE
• An elevated level of this enzyme can be seen in
patient with pernicious anemia.

FEIR, K. ♡
E. CREATINE KINASE (CK)
• Catalyzes: Creatine and ATP into Creatine
Phosphate and ADP (Reversible reaction)
• CK requires Mg and Cysteine; inhibited by excess
Mg
• Pattern of increase in AMI: early enzyme marker G. ISOCITRIC DEHYDROGENASE (ICD)
• Miscellaneous test
• Rises: 4–8 hrs
• This enzyme is found in many tissues, including the
• Peaks: 12 hrs
liver.
• Return to normal: 2–3 days
• An enzyme that catalyzes the conversion of
isocitric acid alpha-ketoglutarate in the Kreb’s
cycle.
• Normal values: 1.2 – 7.0 U/L
• It is moderately elevated in obstructive jaundice
and cirrhosis and is not elevated in myocardial
infarction
• In early stages of viral hepatitis, there is a great
increase in the enzyme
• High values are also seen in neoplastic disease
with metastasis to the liver.

F. ACID PHOSPHATASE
• Can be found in high concentration in prostate
gland secretions and erythrocytes
• Catalyzes: Phosphomonoesters into Alcohol and
Phosphate

FEIR, K. ♡
MT301B – Clinical Chemistry 2 (Lecture) o **They need a substance to pair with para ma
complete.
April 5, 2023 – Types of Enzymes
o **Need ng combination to form other.
• Macroenzymes
o Big in appearance
ENZYME o Conjugated to an immunoglobulin or any
• This are proteins that act as biologic catalyst. large molecule
• An organic catalyst affected by temperature: • Isoenzymes
o 56’c – enzymes are destroyed. o Catalyze the same reaction but differ in terms
o 60’c – precipitation of enzymes occurs. of physical or chemical characteristics and
Question 1: Enzymes are classified into how many types tissue distribution
of reaction? 6 o Example: ALP (4), CK (3), LDH (5)
HILOT
▪ CK: 1 – BB (brain); 2 – MB
Class Type of reaction (myocardium and brain, heart); 3 –
Oxidoreductases Oxidation-reduction MM (muscle)
Transferases Group transfer o **Iso means isa lang → Same action but
Hydrolases Hydrolysis reaction different location or it can be found in other
Lyases Addition or removal of groups to chemical/tissue cell distributions.
form double bonds
Isomerases Isomerization
Factors that affect the function of enzymes
Ligases Ligation of 2 substrates at the
• Temperature
expense of ATP
• Concentration
• pH
Classification of Enzymes • Inhibitors – substances which alter the catalytic action
• Oxidoreductase (dehydrogenase) of the enzyme and consequently slow down, or in
o LDH, G6PD some cases, stop catalysts.
• Transferases (kinase/transaminase) o Competitive
o CK, SGOT/SGPT, GGT o Non-competitive
• Hydrolases o Uncompetitive
o ACP, ALP, LPS, AMS
• Lyase
o Aldolase, Pyruvate decarboxylase
• Isomerase
o Glucose phosphate isomerase
• Ligase
o Glutathione synthetase

Types of Enzymes
• Apoenzyme (incomplete enzyme)
ENZYME-SUBSTRATE (ES) FORMATION
o Protein portion of the enzyme.
• E + S ↔ ES ↔ ES* ↔ EP ↔ E + P
• Holoenzyme (complete enzyme)
• Formed to facilitate transition state.
o apoenzyme + co-factors (non-protein)
o Co-factors: **attached at allosteric site –
pinaka-likod ng enzyme is kidney-bean
shaped, so nasa likod ng kidney-bean
shaped.
▪ Co-enzyme (organic)
▪ Activators (metallic – calcium, zinc)
(non-metallic – chloride)
• Zymogens or Proenzymes
o Inactive form of enzymes.
FEIR, K. ♡
NOTE: hindi siya naka-push through sa intestine
• Enzymes are measured according to activity not by kaya nag-eelevate.
concentration. • The liver cells being diseased cannot synthesize
1. International Unit (IU) – measured micromole adequate amounts of ALP, consequently blood
of substrate per minute levels do not rise much above normal.
**’pag magreread ng enzyme after incubation
of 1-minute, dun pa lang ang initial reading. OBSTRUCTIVE JAUNDICE
Another minute for final reading. • The enzyme that is secreted by healthy liver cells
2. Katal Unit (KU) – measured mole of substrate into the bile, backs up or is regurgitated back to the
per second. liver, thus giving higher levels of enzyme than
**30 seconds lang ‘pag first reading, then normal.
another incubation period of less than 60 • As the liver produces more and excretes more
seconds to 8 minutes. enzyme all the time, because of the obstruction, the
**depending on reagent available in lab. regurgitated or backed up enzyme builds up higher
and higher in the bloodstream giving very high ALP
levels.
A. ALKALINE PHOSPHATASE (ALP)
• **Sa enzymes na ito. enzyme wherein the pH is • Alkaline Phosphatase Isoenzymes:
controlled at alkaline stage o Placenta (P)
• **ALP = diagnosed with liver disease and bone o Intestine (I)
disease. o Liver (L)
• **If the enzyme has elevated result = inflammation or o Bone (B) I Promise to Be
Loyal
problem between normal metabolism and excretion • Identified and separated by:
(liver). 1. Electrophoresis (I-P-B-L) – fastest to migrate is
• The normal serum of ALP is the result of two factors: the liver. 4321
o The rate of release of ALP from tissues,
mainly liver and bone.
o The rate of inactivation and excretion
(depending on the activity)
• The liver is the main source of serum ALP. **sa liver
napproduce ang ALP.
• In liver disease, the increased level comes from
increased release from the liver cell and also from the
regurgitation (**repeat metabolism action) or “backing
up” of the biliary isoenzymes back into the serum. **if
may problem sa liver, abnormal ang formation →
magkakaroon ng back & forth/upper & lower ng ALP
2. Heat denaturation – serum is heated 56C for 10
• ALP is much more greatly elevated in obstructive minutes. (P-I-L-B). 4 3 2 1
jaundice (**obstruction in hepatobiliary duct) than in Pangako Ikaw o Most heat stabile is Placenta – lowest
liver disease. Lang Beh Due to greater rate
and not affected by heat.
of secretion.
o Most heat labile and sensitive is Bone –
HEPATITIS unang nagrereact.
• Since there is damage to the liver cells in liver
disease, the enzyme in the cells is released, giving
elevated ALP but since this enzyme is produced by
the liver cells, the production diminishes, thus
keeping the levels from being extremely elevated.
o **Kaya pag may problem ang liver,
nagnnormalize pa rin ang ALP. Pero kapag
hindi liver ang may problem, bumalik lang
sakanya ang ALP or enzymes because

FEIR, K. ♡
• 2nd marker for AMI
o Rises – 6-8 hrs.
o Peaks – 24 hrs.
o Returns to normal – 5 days
• Methodologies: Reitmen-Frankel (old method) &
Enzymatic method (Malate DH)

2. SGPT/ALT (Serum Glutamate Pyruvate


Transaminase / Alanine Transaminase)
• **Positive yung may darkest color; Negative ang light • Catalyzes: alanine and alpha ketoglutarate into
color glutamate and pyruvate.
• **Ang pag-read ay pataas • Methodologies: Reitman-Frankel & Enzymatic
method (Lactate DH) pusO = asOt
3. Chemical Inhibition (**gagamtt ng chemicals na
hindi sila papalutangin, innegative sila once na
present so hindi sila mageelevate.) NOTE:
Nagao – originate o L-phenylalanine – inhibits placental,
• SGPT and SGOT are both elevated in viral
in placenta. intestine, Regan, and Nagao isoenzymes hepatitis.
o Levamisole - inhibits liver and bone • SGOT is only elevated = concentrated in heart
isoenzymes (AMI)
o Urea – inhibits bone isoenzyme • SGPT is only elevated = liver (fetal) **patay ang
o Leucine – inhibits Nagao isoenzyme liver (mnemonic only) **pwedeng dahil sa
maintenance drugs.
CLINICAL SIGNIFICANCE
)
• Often used in evaluated of hepatobiliary
(obstructive jaundice) and bone disorders.
C. GAMMA GLUTAMYL TRANSFERASE
• Highest elevation of ALP is seen in Paget’s
(GGT)
disease “Osteitis deformans”.
• Used for evaluation of hepatobiliary disease.
PHYSIOLOGIC ELEVATION
• Elevated levels may indicate alcoholism. **we also
• Pregnant and growing children.
considered, drug metabolism – umiinom ba ng
maintenance drugs that may be made of alcohol.
• Catalyzes: Phosphomonoester into alcohol and • Can be used as a means to differentiate the cause of
phosphate. elevation of ALP.
• GGT – enzyme that transfers a y-glutamyl residue to
Reaction name Substrate used an amino acid (transpeptidation). **aalisin sa
Shinowara-Jones-Reinhart Beta-glycerophosphate peptide form ang amino acid para maging simpler into
King-Armstrong Phenylphosphate function.
Bessey-Lowry-Brock (most p-nitrophenylphosphate • This function is involved in peptide and protein
common method in ALP) synthesis.
Bowers-McComb (most p-nitrophenylphosphate • Distribution: Tissue sources of GGT include kidney,
common method in ALP) brain, prostate, pancreas, and liver. Urine contains
significant amounts of GGT.
• Measurement: Total activity is by the SZASZ assay.
B. TRANSAMINASES
In this reaction, the substrate is y-glutamyl-p-
1. SGOT/AST (Serum glutamic-oxaloacetic nitroanilide, with the release of p-nitroaniline.
transaminase / Aspartate Transaminase) CLINICAL SIGNIFICANCE
• SGOT – old name; AST – new name • GGT levels are elevated in almost all hepatobiliary
• Catalyzes: aspartate and alpha ketoglutarate → disorders or biliary tract obstruction as well as in
glutamate and oxaloacetate. patients taking enzyme-inducing drugs (Warfarin,
• It requires Vit. B6 (pyridoxal phosphate) **organic co- Phenobarbital, Dilantin)
enzyme 1. CK 1. OTAS • GGT levels are often examined with ALP levels
2. SGOT 2. PTAL
3. LDH FEIR, K. ♡
o If both GGT and ALP levels are high, some LD3 (HHMM) Lung, Pulmonary embolism,
type of liver disorder is suspected. lymphocytes, lymphocytosis, acute
o If GGT is normal and ALP is high, bone spleen, pancreas pancreatitis
disease is likely present. LD4 (HMMM) Liver, skeletal Hepatic injury or
LD5 (MMMM) muscles inflammation; skeletal
• Patients with acute pancreatitis exhibit increased
muscle injury
GGT levels. In persons with diabetes, GGT is LD6 Alcohol Grave prognosis or
typically increased as triglyceride levels rise. dehydrogenase impending death
(**Directly proportional) • **H – heart; M – muscle
• Because alcohol has enzyme-inducing properties, • **LD2 – kidney ang may problem
the most useful application for GGT is the detection • **Kung may sakit sa puso, 2nd organ affected from
of alcoholism and the monitoring of alcohol intake them is kidney.
by patients during treatment.
• **Check LD6 if comatose na ang patient.

A. 5-NUCLEOTIDASE Specimen Considerations


• Originates in the liver or diseased bone. • No to hemolysis: LDH is 50-100x more concentrated
• 5’-NT becomes elevated in liver disease as does ALP, in RBCs than in plasma.
but 5’-NT does not increase in patients with bone • Do not store in freezing temperature; LD5 is cold
disease. labile, specimens for LDH therefore should be stored
• Serum levels of 5’-NT are more sensitive to liver at room temperature.
cancer.
• **This confirms for liver cancer. Methodologies
• An elevated enzyme level of 5-nucleotidase is a • Wacker (Forward reaction)
sensitive marker for what disease? Liver Carcinoma o Lactate → Pyruvate
• Wrobleuski La Due (Reverse reaction) – product
ang magsstart papunta kay lactate) → most preferred
Test Liver Bone is reverse
ALP ↑ N o Pyruvate → Lactate
GGT ↑ N • Highest elevation of LD can be seen in pernicious
Leucine ↑ N anemia.
aminopeptidase • Liver disease with the highest elevation of LD is toxic
5’NT ↑ N hepatitis. **viral hepatitis – AST/ALT elevation

B. LACTATE DEHYDROGENASE C. ISOCITRIC DEHYDROGENASE (ICD)


• An elevated level of this enzyme can be seen in • Miscellaneous test
patient with pernicious anemia (**problem in • This enzyme is found in many tissues, including the
absorption of Vit. B12) liver.
• Late marker (3rd) for Acute Myocardial Infarction • An enzyme that catalyzes the conversion of isocitric
(AMI): acid alpha-ketoglutarate in the Krebs’s cycle.
o Rises: 12–24 hrs.
• Normal values: 1.2 – 7.0 U/L
o Peaks: 48–72 hrs. (up to 3 days)
• It is moderately elevated in obstructive jaundice and
o Return to normal: 10 days
cirrhosis and is not elevated in myocardial infarction.
• Catalyzes: Lactate to pyruvate
• In early stages of viral hepatitis, there is a great
• Has 5 isoenzymes
increase in the enzyme.
o Electrophoresis: LD 2, 1, 3, 4, 5
• High values are also seen in neoplastic disease with
o LD flip: LD1>LD2 (Associated with AMI but
metastasis (**kumakalat na to other organ ang sakit)
not specific)
to the liver.
Isoenzyme Tissue source Disorder
LD1 (HHHH) Heart, RBC MI, Pernicious anemia
LD2 (HHHM) Renal cortex Acute renal infarction

FEIR, K. ♡
D. ALDOLASE • Catalyzes: Phosphomonoesters into Alcohol and
• Involved in the metabolism of carbohydrates in tissue. Phosphate
(**we need to check if normal pa ang carbohydrate
metabolism in liver.)
• Increased levels are found in acute hepatitis,
myocardial infarction, and muscular dystrophy.
• It is normal or slightly elevated in obstructive jaundice.

E. IRON (for liver testing)


• Serum iron levels are increased two to three times
in hepatitis.
• Cirrhosis – lower than normal serum iron levels.
• Obstructive jaundice – normal iron levels.

♡♡♡♡♡♡♡♡♡♡♡♡♡♡♡♡♡♡♡♡♡♡♡♡♡♡♡♡♡♡♡♡♡♡♡♡

F. CREATINE KINASE (CK)


• Catalyzes: Creatine and ATP into Creatine
Phosphate and ADP (Reversible reaction)
• CK requires Mg and Cysteine; inhibited by excess Mg
• Pattern of increase in AMI: early enzyme marker
• Rises: 4–8 hrs
• Peaks: 12 hrs
• Return to normal: 2–3 days

Isoenzymes
• CK1 – CKBB – Brain
• CK2 – CKMB – Heart
• CK3 – CKMM – Muscle

Specimen Considerations and Patient Preparation


• No to hemolysis, CK is not present in RBCs.
However, adenylate kinase, which catalyzes a
somehow similar reaction as that of CK.
• Stored in the dark since it has been found that CK is
inactivated by light. (Bishop 6th ed.)

Methodologies
• Tanzer Gilvarg method (Forward reaction)
o Creatine → Creatine Phosphate
• Oliver Rosalki method – Rosalki Hess (Reverse
reaction)
o Creatine Phosphate → Creatine

G. ACID PHOSPHATASE
• Enzyme that elevates as a marker for prostatic
carcinoma.
• Can be found in high concentration in prostate gland
secretions and erythrocytes.

FEIR, K. ♡
MT301B – Clinical Chemistry 2 (Lecture) NOTE:
• Amylase is activated by calcium ions.
April 5, 2023 (cont.) – Enzymes
• Most enzymes require metals as activators or
cofactors.
• CK and ALP require Mg+2 for full activity, and
ENZYMES amylase requires Ca+2.
• Enzymes have different long chain of amino acids → • Metals required for activity should be components
they form a different group, sizes (micro and macro), of the substrate used for enzyme analysis.
and action.
• The substrate must also contain anions required
(e.g., Cl– for amylase) and should not contain
A. AMYLASE (AMS)
inhibiting cations or anions (e.g., Zn+2 and Mn+2
• This enzyme is considered the smallest of all for CK).
enzymes. (I AMSmall)
• Requires Calcium and Chloride (cofactors → metal
and non-metal) B. LIPASE (LPS)
• Enzyme that catalyzes the breakdown of starch and • An enzyme that hydrolyzes the ester linkages of fats
glycogen and form maltose. to produce alcohols and fatty acids. **pantunaw ng
o Polysaccharide to monosaccharides taba sa katawan
• **Carbohydrates lang ang target. • **Amylase and Lipase are two enzymes responsible
• Product: (**monosaccharide) glucose, maltose, and for pancreatic enzyme/action.
dextrin “bangungot”- yung tao ay hindi nagising • **Lipase – iba’t-ibang lipids. Para mabreakdown si
• Marker for Acute pancreatitis (**”titis” – lipids sa katawn, need ng lipase. End product is fatty
inflammation) acid → nagiging glucose.
• Marker for Acute pancreatitis.
Isoenzymes o **Between amylase and lipase, the most
• Salivary AMS – nasa saliva, salivary gland sensitive enzyme marker is lipase. Kasi mas
• Pancreatic AMS – in pancreas concentrated sa pancreas.
• **In testing, no pipetting of mouth dahil may masama
na laway mo sa reagent → false elevation of enzyme 3 Forms of LPS
result. • Pancreatic lipase (pancreas)
o **Para hindi mapuno ng taba ang pancreas.
Methodologies At the same time, ang hindi pa name-
• Saccharogenic (Somogyi) metabolize ng liver ay dadalhin sa pancreas
o Measures the product formed – maltose. para ma-metabolize ng pancreatic lipase.
o Measures reducing sugars produced by o **pag maraming kinain, marami rin siyang id-
hydrolysis (**split up) of starch. digest.
• Amyloclastic (Iodometric) • Colipase (is cleared by the kidney) decrease GFR
o Measures the amount of substrate left o ** ↑ fats metabolism = ↓ Glomerular filtration
(starch) which forms a blue iodine-starch rate
color. • Lipoprotein lipase
o Amylase activity by following decrease in o **Found in vascular membrane (arteries,
substrate concentration. (**mamemeasure veins) → kaya pag super dami ng fats,
lang si amylase pag bumaba ang substrate pinupunta sa arteries “thrombophlebitis”
conc.)
• Chromogenic Methodologies
o Measures the increasing color from • Cherry and Crandall
production of product coupled with a o Old method for lipase.
chromogenic dye. o Measures how much fatty acid is produced.
• Continuous monitoring o Serum is incubated with an olive oil.
o Coupling of several enzyme systems to ▪ Medyo matagal i-convert ni olive oil
monitor amylase activity. si fatty acids. Need muna i-read after
o **Incubate muna then after 1-minute saka 24 hours.
irread. 3-4 readings and need.

FEIR, K. ♡
o Emulsion (the substrate) and the amount of Babson a-naphthyl phosphate
olive oil converted to fatty acids is measured and Reed For continuous monitoring
colorimetrically. (Triolein) NEW IMBES NA (Most
Thymolphthalein
OLIVE OIL Roy specific for
monophosphate
ACP)
• Turbidimetric method
Fluorescent,
o Which an olive oil substrate is used and the
Reitz, 4- more
measurement of the decreased. Guilbault methylumbelliferonephosphate improved
o Amount of substrate is made. sensitivity
o The amount of substrate used is measured
instead of product formed.
o The more substrate use, the clearer or less D. CREATINE KINASE MB (CK-MB)
turbid the test becomes. • This enzyme is the first marker that elevates in
▪ **Turbid at start, but magiging clear myocardial infarction.
sa reaction. • **Enzyme markers for AMI: CK → AST → LDH
▪ **Mas turbid - disadvantage • It belongs to CK enzyme that requires magnesium
o Simpler and more rapid; measurement of rate and cysteine. **needs co-factor para mag-proceed sa
of clearing as an estimate for lipase activity. action.
• Colorimetric method
o Coupled reactions with peroxidase or glycerol Isoenzymes
kinase. • CK1 – CKBB (CVA Cerebrovascular accident) →
fastest to migrate
• CK2 – CKMB (MI) → naiiwan sa electrophoresis
C. ACID PHOSPHATASE (ACP) • CK3 – CKMM (Muscle injury) → naiiwan sa
electrophoresis; Muscle Atrophy (lumiliit ang muscle
• Enzyme that elevates as a marker for prostatic
carcinoma (screening only). and nawawalan ng function)
• Elevation of all isoenzymes → Duchenne’s
• High in concentration in prostate gland and
erythrocytes. Muscular Dystrophy
• Rises – 4-8 hrs.
• **False elevation = hemolyzed
• **’Pag may request na ACP, mostly ang patients ay • Peaks – 12 hrs.
male since prostatic carcinoma • Returns to normal – 2-3 days
• **Confirmation – Prostate Specific Antigen; Other Specimen considerations
lab testing: 3 vial collection; Ultrasound & Ct scan • No to hemolysis and stored in the dark because it is
o ACP → PSA → 3 vial → Ultrasound → CT inactivated by light. (Bishop 6th ed.)
scan
Methodologies
Isoenzymes • Tanzer Gilvarg method (Forward reaction)
• Erythrocytic ACP (fastest to migrate) o Creatine + ATP → Creatine phosphate +
• Prostatic ACP Differentiated in ADP
electrophoresis.
Method • Oliver Rosalki (Reverse reaction) (Reference
method – 6x faster than Tanzer Gilvrg)
• Babson and Reed
o Creatine phosphate + ADP → Creatine +
• Roy (reference method)
ATP
Reaction
Substrate used Comments NOTE:
name
Lengthy • Troponin T – cardiac marker that consistently
Bodansky Beta-glycerophosphate assay, non- increased in persons who exhibit unstable angina.
specific • Persons with unstable angina (angina at rest) who
Gutman, have an elevated TnT or TnI are at eight times
King- Phenylphosphate Non-specific greater risk of having an MI within the next 6
Armstrong months.
Rapid, non-
Hudson p-nitrophenylphosphate • This property is being used to identify short-term
specific
risk patients who should be considered for
coronary angioplasty. The reference range for
FEIR, K. ♡
troponin is very low (0–0.03 ng/mL); persons with o In the RBC, G6PD functions to maintain
unstable angina usually have values between 0.04 NADPH in a reduced form to protect
and 0.1 ng/mL without clinical evidence of AMI. hemoglobin from oxidation and to prevent
• CK-MB and myoglobin have not been useful in red cell hemolysis.
identifying persons with unstable angina.
Measurement
• The formation of NADPH is measured
E. CHOLINESTERASE colorimetrically.
• An enzyme that hydrolyzes the esters of choline.
Diagnostic significance
• 2 types:
• Clinically deficiency of G6PD is an inherited sex-
o Ache (Acetylcholinesterase)
linked trait most common in African-Americans.
▪ Substrate: acetylcholinesterase
o **Carrier of link is mother, and ipapasa niya
▪ Found in RBCs, brain, and nerve
sa son (opposite sex).
cells.
• Increases of G6PD are seen in persons with MI and
o pChe (pseudocholinesterase) **pseudo
Megaloblastic anemia (**size of RBC are bigger).
means false
▪ Substrate: butyryl esters
▪ Found mostly in the serum, liver,
pancreas, heart, and white matter of SUMMARY:
the nervous system. • Specimen of choice (Enzymes: serum)
• NOTE: no hemolysis dahil may presence of RBC. • No to hemolysis
• Always take into considerations the regulated
Measurement temperature of the specimen to be analyzed based
• Manometric techniques – measure liberated carbon on the requested test.
dioxide from the formation of acetic acid. • Be familiarized with the method of the analyzer that
• Electrometric measures – determine enzyme you are operating.
activity by measuring the pH decrease resulting from • Quality control must Always observe and work with
liberation of acetic acid. presence of mind for an accurate and precise
• Photometric method (Ellman technique) – the result.
substrate is a thiol ester that produces a thiol, which • Always bear in mind that Physician and Patients
reacts with a disulfide to form a colored compound. rely on our result because we are the backbone of
• **all enzymes elevated ‘pag may abnormal the Clinical Diagnosis.
metabolism/problem sa organ except cholinesterase.
Lower ang level niya kapag may pesticide poisoning.

F. G6PD
• Glucose 6-phosphate dehydrogenase is an
oxidoreductase that catalyzes the oxidation of G6P
to 6-phosphogluconate which is an important first
step in the pentose-phosphate shunt of glucose
metabolism. **Kaya tinawag na glucose 6-
phosphate dehydrogenase kasi importante siya sa
glucose metabolism by catalyzing G6P to 6-
phosphogluconate.
• Distribution:
o Tissue sources of G6PD include the adrenal
glands, spleen, thymus, RBCs, and lymph
nodes.
▪ **Important sa RBC because
nagcciruclate ang RBC papunta sa
liver para tumulong sa glucose
metabolism.

FEIR, K. ♡
MT301B – Clinical Chemistry 2 (Lecture) • In multiple myeloma, synthesis of large quantities of
monoclonal immunoglobulin by plasma cells often
April X, 2023 – Liver Enzyme Test (Lab.)
results in decreased synthesis of albumin.
• In glomerulonephritis and nephrotic syndrome, both
total protein and albumin are low owing to loss of
TEST FOR LIVER FUNCTION proteins through the glomeruli.
1. Proteins • In hepatic cirrhosis, decreased hepatic production of
2. Liver Enzymes protein results in low total protein and albumin.
3. Ammonia Bilirubin
4. Bilirubin
CLINICAL CASE
A. PROTEINS Chronic Decrease in albumin and increase in
• Method of Protein Analysis: liver gamma globulin, beta globulin
o Refractive Index cirrhosis **Can develop into liver cancer
o Biuret Method – most commonly used **The normal is albumin is greater than
method globulin. But once may decrease sa
▪ Principle: Whenever an alkaline albumin, and increase in globulin, there
copper reagent acts with substances is a disease affecting liver.
containing two or more peptide Obstructive Albumin is slightly decrease and
bonds, it produces a blue violet color. jaundice gamma globulin is slightly increased
o Turbidimetric Chronic Decreased albumin and increased
o Electrophoresis hepatitis/ globulins
Cirrhosis
Question 1: Upon which principle is the biuret method
based? Coordinate bonds between Cu+2 and carbonyl
and imine groups of peptide bonds. B. LIVER ENZYMES

Question 2: Which of the following protein methods has


the highest analytical sensitivity? Folin-Lowry C. AMMONIA
• The Folin–Lowry (Lowry's) method uses both biuret • **Test to determine toxification function of liver
reagent and phosphotungstic and molybdic acids to • Specimen considerations:
oxidize the aromatic side groups on proteins. o Serum is NOT used; EDTA plasma or
• The acids oxidize the phenolic rings of tyrosine and heparinized plasma can be used.
tryptophan. These, in turn, reduce the Cu+2 in the o Placed immediately on ice bath
biuret reagent, increasing sensitivity about 100-fold. o Analyzed immediately (20 minutes)
o No to hemolysis
Question 3: Kjeldahl's procedure for total protein is based • Clinical significance: Reye’s syndrome (correlated
upon the premise that: The nitrogen content of proteins with CSF – Glutamine test)
is constant. • Methodologies:
o Conway micro-diffusion
TP A/G
o Ion-exchange
• Total Protein – Biuret Method o Coupled enzyme
• Albumin – Dye Binding Method o ISE
o BCG – binds to albumin producing a color
that is read spectrophotometrically
o BCP – may also be used
D. BILIRUBIN
• A/G ratio
o Normal ratio is albumin > globulin.
• Bilirubin methods:
o Reported in grams/dL o Icterus index – 0.01% K2Cr2O7
o Any disturbances will lead into albumin < o Ehrlich (1883) – described the reaction of
globulin (nephrosis, multiple myeloma) bilirubin with diazotized sufanilic acid to
form a blue color in strongly acid or alkaline
Question 4: High serum total protein but low albumin is solutions.
usually seen in: Multiple myeloma
FEIR, K. ♡
o Van den Bergh used this reaction to • Serum or plasma may be used.
determine serum bilirubin using alcohol. • A fasting specimen is preferred to avoid
interference from lipemia.
Question 5: Which reagent is used in the Jendrassik-
• Total bilirubin is measured by adding caffeine to the
grof method to solubilize unconjugated bilirubin?
specimen to which diazotized sulfanilic acid is
Caffeine
added.
• Evelyn and Malloy used 50% methanol instead of • The conjugated and unconjugated bilirubin both
ethanol which prevented precipitation of CHONs. react with the diazo reagent which produces
(purple color) azobilirubin.
• Jendrassik and Grof method used the diazo • To this ascorbic acid, alkaline tartrate, and dilute
technique, but speed it up by the use of Caffeine hydrochloric acid (HCl) are added, producing a
and Na benzoate (blue/green) blue azobilirubin solution.
• The conjugated bilirubin is measured by acidifying
the sample with dilute HCl.
SERUM BILIRUBIN
• To this is added diazotized sulfanilic acid which
• Turbidity due to lipemia and hemolysis affect the produces azobilirubin.
bilirubin values.
• The reaction is stopped by adding an ascorbic acid
• Lipochrome pigments, such as carotene from solution.
carrots are detected in bilirubin procedures.
• Alkaline tartrate is then added, along with caffeine
• Bilirubin pigments are destroyed by light. reagent, which results in the production of a blue
• Specimens should be fasting, free from hemolysis azobilirubin solution.
and protected from light. • Reference values:
• Hemolysis causes falsely low results with the diazo o TB – 0.2 to 1.0 mg/dL
method. o DB – 0 to 0.2 mg/dL
• Direct sunlight at room temperature (also o IB – 0.2 to 0.8 mg/dL
Fluorescent lights) can cause up to 50% decrease • Conversion factor: mg/dL to umol/L (17.1)
in bilirubin within one hour. • Critical value: > 18mg/dL

EVELYN & MALLOY


• Bilirubin in the serum is coupled with diazotized
sulfanilic acid to form azobilirubin.
• The intensity of the color formed is proportional to
the amount of bilirubin present in the serum.
• The conjugated (direct) bilirubin is soluble in water,
while the unconjugated (indirect) is soluble in
alcohol.
• Principle: In acid solution, sodium nitrite gives
nitrous acid, this acid reacts with sulfanilic acid and
forms a diazonium salt which reacts with bilirubin
to form isomers of azobilirubin (purple color)

JENDRASSIK & GROF


• Used the diazo technique, but speed it up by the
use of caffeine and sodium benzoate.
• The final color is blue or green, because it is
developed at a strongly alkaline pH (modification of
Evelyn and Malloy).
• This procedure is the procedure of choice.

FEIR, K. ♡
MT301B – Clinical Chemistry 2 (Lecture) FUNCTIONS
• Volume and osmotic regulation. (Na, Cl, K)
April X, 2023 – Electrolytes
• Myocardial rhythm and contractility. (K, Mg, Ca)
• Cofactors in enzyme activation. (Mg, Ca, Zn)
• Regulation of ATPase; ion pumps. (Mg)
ELECTROLYTES
• Acid-base balance. (HCO3, K, Cl)
• A “medical/scientific” term for salts, specifically ions.
• Blood coagulation. (Ca, Mg)
• The term electrolyte means that this ion is electrically
• Neuromuscular excitability. (K, Ca, Mg)
charged and moves to either a negative or positive
• Production and use of ATP from glucose (Mg, PO4)
electrode:
o Ions that move to the cathode (cations) are
caTions → posiTive positively charged.
NaK MaG ZaiNg Ca METHODS
aNions → Negative ▪ Cations (Na, K, Mg, Ca, and Zn)
o Ions that move to the anode (anions) are 1. FLAME EMISSION
negatively charged. SPECTROPHOTOMETRY (FES)
▪ Anions (Cl, HCO3) • In flame emission spectrometry, the sample solution
• Fluid always contains equal numbers of cations and is nebulized (converted into a fine aerosol) and
anions. introduced into the flame where it is desolvated,
• TBW (Total Body Water) makes up about 60% of vaporized, and atomized, all-in rapid succession.
male body weight and 50% of female weight. • Subsequently, atoms and molecules are raised to
• 40-75% is the average water content of the human excited states via thermal collisions with the
body. constituents of the partially burned flame gases.
• 60% of the body’s water is inside cells, and the rest is • Upon their return to a lower or ground electronic state,
in the bloodstream or tissue fluids. the excited atoms and molecules emit radiation
• About 3 Liters of fluid passes from the blood to the characteristic of the sample components.
tissue spaces daily. • The emitted radiation passes through a
monochromator that isolates the specific wavelength
2 main compartments for the desired analysis.
• Two thirds of TBW is in the intracellular compartment • A photodetector measures the radiant power of the
and the other third is ECF. selected radiation, which is then amplified and sent to
• Electrolytes are ions that exist in all of these fluids, but a readout device, meter, recorder, or microcomputer
the concentration varies depending on individual’s ion system.
and compartment.

Extracellular Fluid Intracellular Fluid


(ECF) (ICF)
• 25% of the ECF is • Fluid within cell
plasma.
• 2 sub compartments:
A. Interstitial body
fluid
o 15-20%
TBW
o Normal
plasma: 93%
water + 7%
solutes
B. Intravascular
(plasma)
o 5-10% TBW

FEIR, K. ♡
o The disc rotated at high constant speed,
when the mirrored quarter in front of the
lamp, it reflects the radiation.
o The second moment the open in front of the
lamp and the radiation passes to the sample
being absorbed by it and reaches the
detection impulses.
o The detector converts the radiation to
Sedico-flame photometer alternating current signal and amplified it.
• Atomizer
o A flame atomizer
o It is a graphite furnace heated electrically up
to 6OOO’c and contains a ribbon or boat in
which one can inject the sample.
o Upon heating the furnace
o The sample is asher, then atomized by
action of heat.
• Monochromator
o Grating type monochromators are used.
Applications
o Types:
• Determination of Na, K, Ca, and Mg in biological fluids ▪ Diffraction gratings
like serum, plasma, urine etc. ▪ Transmission gratings
• Soil samples are analyzed mainly for Na, K, etc. by • Detectors
flame photometry. o Used are:
• Estimation of alkali-alkaline earth metals. ▪ Photo tube
• Determination of metals like Pb, Mn in petroleum ▪ Photo multiplier tube
products and organic solvents. • Read Out Device/Meter
o The read-out device is capable of displaying
2. ATOMIC ABSORPTION the absorption spectrum as well as the
SPECTROPHOTOMETRY (AAS) absorbance at the specified wavelength.
• Atoms in the vapor state are subjected to external o Beer’s law is obeyed in wide concentration
sources of radiation which produces one line or beam range technique.
of monochromatic light with single wavelength.
• This wavelength is a resonance one for the atoms that Application
will be absorbed by them. • Estimation of trace elements in biological fluids like
• All elements can be determined by this technique. blood, urine, etc.
• Estimation of elements like Copper, Zinc etc in food
Instrumentation products.
• Source of radiation • Estimation of Mg, Zn in blood.
o Hollow cathode lamp – a tube with a front • Estimation of Hg in thiomersol solution.
quartz window contains an anode of
Tungsten and a cylindrical cathode, the
material of which is the same element of the
sample to be determined.
o The glass tube is filled with neon or argon at
a pressure of 1 to 5 tore. When a low
potential is applied between the two-
electrode ionization of the gas occurs, ions
move rapidly to the cathode.
• Chopper
o Its function is to fluctuate the source output.
o It is a circular disc divided into four quarters,
two are mirrored and two are opened.

FEIR, K. ♡
o Standard solutions are prepared by serial
dilution of a concentrated standard.
o The recommended Ionic Strength
Adjustment Buffer, (ISAB), is added to each
standard as well as to the unknown samples.
o The system is calibrated, and the information
stored in the microprocessor memory.
o The electrode potential of each of the
unknown solutions is then measured and the
concentration of the ion read directly from the
meter.
• There are four types of ion selective electrode whose
construction and mode of operation differ
considerably.

4 types
• Glass body electrode
o The most common ISE is the glass-bodied
pH combination electrode.
o The sodium (Na+) combination has a similar
construction which houses a glass bulb that
is sensitive to sodium ions in solution.
• Solid state (crystalline membrane)
o The electrode potential of standard and
sample solutions is measured across a solid,
polished crystalline membrane.
o The crystalline material is prepared from a
Atomic Absorption single compound or a homogeneous mixture
Flame Photometry of compounds (for example, the fluoride ISE
Spectroscopy
has a Lanthanum Fluoride crystal).
Radiation emitted by Radiation absorbed by
exited atoms ground state atoms • Liquid ion exchange (polymer membrane)
o These electrodes use a replaceable
Temperature dependent Temperature independent
membrane cap which has a solid, polymeric
Linear over narrow range Linear over wide range of
membrane containing a selective ion
concentration concentration
exchanger.
Few elements Wide range of elements
o The electrode potential of solutions is
measured by their effect on the ion exchange
3. ION SPECIFIC SELECTVE ELECTRODE material.
(ISE) o Due to the complex properties of the ion
• Most commonly/routinely used method in clinical exchangers, they are subject to more
laboratory. interferences than other ion selective
• An ion selective electrode generates a difference in electrodes.
electrical potential between itself and a reference • Gas sensing type
electrode. o Electrodes, including the ammonia ISE, use
• The output potential is proportional to the amount or a gas sensing mode of operation.
concentration of the selected ion in the solution. o In the case of ammonia, a caustic solution is
• The concentration is a measure of the number of ions added to the sample solution to liberate
in a specific volume. ammonia.
• Direct potentiometry o The gas permeates through a membrane
o This is the simplest and most widely used and changes the pH of the filling solution. The
method of obtaining quantitative results using change in pH is proportional to the ammonia
Ion Selective Electrodes. concentration.

FEIR, K. ♡
o This gives a quantitative measurement of the REGULATION
ammonia in the sample solution. • The plasma Na+ concentration depends greatly on
the intake and excretion of water.
Methods of analysis
• Regulation is performed by the renal system in two
• There are a number of methods of analysis with ion
ways.
selective electrodes.
• Microprocessor based instruments are programmed
1. RENIN-ADH SYSTEM
with direct concentration modes of operation.
• Low blood volume (as in cardiac failure) induces
• Calibration and sample measurement are carried out
secretion of renin, a vasoconstrictor, from the kidney,
automatically.
which raises blood pressure and causes production
• The traditional drawing of calibration graphs is no
of ADH. In turn, fluid volume is increased by the
longer necessary.
retention of sodium.
Other methods 2. RENIN-ALDOSTERONE SYSTEM
• One point calibration • Low blood volume induces secretion of renin, which
• Incremental techniques induces production of aldosterone by the adrenal
• Multiple sample addition glands. In turn, kidney reabsorption of sodium and
• Titrimetric procedures retention of water increase.

♡♡♡♡♡♡♡♡♡♡♡♡♡♡♡♡♡♡♡♡♡♡♡♡♡♡♡♡♡♡♡♡♡♡♡♡ 3 PRIMARY IMPORTANT PROCESS


• The intake of water in response to thirst, as stimulated
A. SODIUM (Na+) or suppressed by plasma osmolality.
• Most abundant extracellular cation, representing 90% • The excretion of water, largely affected by AVP
of all extracellular cations and largely determined the (Arginine vasopressin hormone) release in response
osmolality. to changes in either blood volume.
• Maintains the volume of ECF and cell size and shape. • Blood volume status, which affects Na+ excretion
• When combined with chloride, the resulting through aldosterone angiotensin II and ANP (Atrial
substance is table salt. Excess sodium (such as that Natriuretic peptide).
obtained from dietary sources) is excreted in the NOTE:
urine. Sodium regulates the total amount of water in • Normally, 60-70% of filtered Na+ is reabsorbed in
the body and the transmission of sodium into and out the proximal tubule; electroneutrality is maintained
of individual cells also plays a role in critical body by either Chloride (Cl-) reabsorption or Hydrogen
functions. ion (H+) secretion.
• (A significant part of water regulation in the body since
water goes where the sodium goes.)
• Essential for transmitting nerve impulses. (Electrolyte CLINICAL APPLICATIONS
that helps with electrical signals in the body, allowing
muscles to fire and the brain to work.) 1. HYPONATREMIA
• Many processes in the body, especially in the brain, • Defined as a serum/plasma level < 135 mmol/L.
nervous system, and muscles, require electrical Levels below 130 mmol/L are clinically significant.
signals for communication. The movement of sodium • Low serum sodium caused by gastrointestinal loss,
is critical in the generation of these electrical signals. burns or renal problems.
Therefore, too much or too little sodium can cause • Dilutional hyponatremia is a relative decrease of
cells to malfunction, and extremes in the blood sodium caused by excess body water, as in nephrotic
sodium levels (too much or too little) can be fatal. syndrome or cirrhosis.
• A normal plasma osmolality is approximately 295
Conditions
mmol/L, with 270 mmol/L being the result of Na+ and
• Decreased level maybe caused by increase Na loss.
associated anions.
• Decreased water retention or water imbalance.
• A Normal blood sodium level is 135-145
• Increased Na loss in the urine can occur with
milliequivalents/liter (mEq/L), or in international units,
decreased aldosterone production, certain Diuretics
135-145 millimoles/liter (mmol/L).
(thiazides) with Ketonuria (Na loss with Ketones) or
salt-losing nephropathy (with renal tubular disorder)
FEIR, K. ♡
• Prolonged vomiting or diarrhea or severe burns can Functions
result in Na loss. • Regulation of neuromuscular excitability
• Contraction of the heart
2. HYPERNATREMIA • ICF volume
• Serum/plasma level > 145 mmol/L • Hydrogen ion concentration
• Usually occurs when water is lost through diarrhea,
excessive sweating, or diabetes insipidus, and when CLINICAL APPLICATIONS
sodium is retained through acute ingestion,
hyperaldosteronism, or infusion of hypertonic 1. HYPOKALEMIA
solutions during dialysis. • A lower than normal difference increases cell
• (most commonly involve CNS as a result of excitability, leading to muscle weakness.
Hyperosmolar state). • A result of decreased dietary intake, gastrointestinal
• Treatment: Fluid administration and monitoring loss, or renal dysfunction can produce irregular
during treatment. heartbeat.
• It occurs when the body losses too much potassium
DETERMINATION OF Na+ from causes like vomiting, diarrhea, sweating, and
medications such as diuretics or laxatives.
• Specimen: serum, plasma, and urine are all
acceptable for Na+ measurements. • It is often seen in diabetic ketoacidosis, where
potassium is excessively lost in urine.
• When plasma is used: Lithium heparin, NH4
heparin, and lithium oxalate are suitable • Since chemicals in the body are related in their
anticoagulants. metabolism, low magnesium levels can be associated
with hypokalemia.
• Urine (24hrs. collection): 40-220 mEq/L
2. HYPERKALEMIA
METHODS • Severe hyperkalemia can ultimately cause a lack of
• Flame Emission Spectrophotometry (FES) muscle excitability, which may lead to paralysis or
• Atomic Absorption Spectrophotometry (AAS) fatal cardiac arrhythmia.
• Ion Specific Selective Electrode (ISE) – most • High potassium levels are most often associated with
commonly/ routinely used method in clinical lab. kidney failure, in which potassium levels build up and
cannot be excreted in the urine. Emergency dialysis
is often required to remove the potassium.
B. POTASSIUM (K+) • It occurs following excessive dietary intake, adrenal
• Is the major intracellular cation in the body, with a failure, blood transfusions or crush injuries.
concentration 20 times greater inside the cells than
outside. This is maintained by the Na pump, 3 FACTORS THAT INFLUENCE THE DISTRIBUTION
(exchanges 3 Na for 1 K) OF K+ BET. CELLS AND ECF ARE:
• Only 2% of potassium is in the plasma. • Potassium loss frequently occurs whenever the Na, K
• Potassium normal values ATPase pump.
o Serum: 3.5- 5.3 mEq/L • Insulin promotes acute entry of K+ ions into the
o Newborns: slightly higher 3.7-5.9 mEq/L skeletal muscle and liver by increasing Na, K ATPase
o Urine (24hrs. collection): 25-125 mEq/L activity.
• The K+ ion concentration has a major effect on the • Catecholamines, such as epinephrine (B2-
contraction of skeletal and cardiac muscles. stimulator), promote cellular entry of K+ whereas
• K+ concentration also affects hydrogen- ion propranolol (B-locker) impairs cellular entry of K+.
concentration in the blood.
o As K+ ions are lost from the body, sodium
COLLECTION OF SAMPLES
and hydrogen ions move into the cell. The
• Proper collection and handling of samples for K+
hydrogen-ion concentration is therefore,
analysis is extremely important.
decreased in the ECF, resulting in alkalosis)
o The coagulation process releases K+ from
• Exercise promotes released of K+ ion from cells
platelets. If the patient’s platelet count is
which may increase plasma K+ by 0.3-1.2 mmol/L.
elevated(thrombocytosis), serum K+ maybe
with mild to moderate exercise and by as much as 2
further elevated.
to 3 mmol/L with exhaustive exercise.
FEIR, K. ♡
• If a tourniquet is left on the arm too long during blood • Caused by:
collection or if patients excessively clench their fists o Dehydration
or otherwise exercise their forearms before o Acute renal failure
venipuncture, cells may release K+ into the plasma. o Prolonged diarrhea with loss of sodium
• Whole blood samples for K+ ion determination should bicarbonate and salicylate intoxication
be stored at room temperature.
2. HYPOCHLOREMIA
• Hemolysis – must be avoided because of the high
• Low serum chloride.
K+ content of erythrocytes.
• Decrease Cl- level.
• Heparin – anticoagulant of choice.
• Occur with excessive loss of Cl- from prolonged
vomiting, diabetic ketoacidosis, aldosterone
METHODS
deficiency or salt-losing renal diseases such as
• ISE measurements. (Valinomycin membrane)
pyelonephritis.
• Flame photometry
DETERMINATION OF Cl-
REGULATION • Specimen: Serum or plasma maybe used with lithium
• Diet – easily consumed (bananas, etc.) heparin being anticoagulant of choice.
• Kidneys – potassium is readily excreted but gets • Urine: 24hrs urine collection.
reabsorbed in the proximal tubule under the control of • Sweat also suitable for analysis.
Aldosterone.
METHODS
• ISE – that use a semipermeable membrane to
develop a potential between two different ion
C. CHLORIDE (Cl-) concentrations.
• Major extracellular anion that acts to maintain osmotic • Amperometric-coulometric titration – in which
pressure, keeps the body hydrated, and maintains silver ions are combined with chloride; when excess
electric neutrality via interaction with sodium or free silver ions are noted, elapsed time is relative to
carbon dioxide. the chloride concentration.
• It usually follows Na (if one is abnormal, so is the • Mercurimetric titration and colorimetry
other)
• Spectrophotometric mtd- uses ferric perchlorate
NOTE: that reacts to form a colored compound.
• Regulation of Chloride depends on dietary
intake, sodium concentration, and the chloride
shift, which moves chloride into RBCs as D. BICARBONATE
bicarbonate diffuses out to produce • Second most abundant anion in the extracellular fluid.
electroneutrality. It is a major component of the blood buffering system,
• Chloride disorders are often a result of the same accounts for 90% of total blood carbon dioxide, and
causes that disturb Na+ levels because Cl- maintains charge neutrality in the cell.
passively follows Na+. • Maintain acid-base balance.
• Specimen: Serum or lithium plasma

Functions Clinical Applications


• Maintaining osmolality • Acid-base imbalances cause changes in bicarbonate
• Blood volume and CO2 levels.
• Electric neutrality • CO2 is a waste product. Continuously produced as a
result of cell metabolism.
• Decrease bicarbonate may occur from metabolic
CLINICAL APPLICATIONS
acidosis as HCO3 combines with H+
1. HYPERCHLOREMIA • To produce CO2 which is exhaled by the lungs. The
• Elevated serum chloride. typical response to metabolic acidosis is
• Increase Cl- in the blood. compensation by hyperventilation which lowers
• Occur when there is an excess loss of HCO3- ion as a PCO2.
result of GI losses, RTA, or metabolic acidosis.

FEIR, K. ♡
• Increase total CO2 concentrations occur in metabolic • Reference ranges:
alkalosis as HCO3 retained, as a result of o Total Ca (adults): 86-10.3 mg/dL
compensation by hypoventilation. o Free Ca (adults): 4.6-5.3 mg/dL

3 HORMONES RESPONSIBLE FOR Ca


E. MAGNESIUM REGULATION
• Fourth most abundant cation in the body and second • Parathyroid hormone
most abundant intracellular ion. • Vitamin D
• Approximately 53% of Mg in body found in bone, 46% • Calcitonin
in muscle and other organs and soft tissue, and less
NOTE:
than 1% is present in serum and RBC.
• About 99% of Ca in the body is part of the bone.
• The magnesium level is Regulated by the kidneys
The remaining 1% is mostly in the blood and other
through reabsorption and excretion.
ECF.
• PTH enhances reabsorption by the kidneys and
• Forms of Ca in the blood:
intestinal absorption.
o 45% circulates as free Ca ions (ionized Ca)
• Rich sources of Mg in the diet include raw nuts, dry
o 40% bound to protein (mostly protein)
cereal and hard drinking water; other sources include
o 15% bound to anions (such as HCO3,
vegetables, meats, fish and fruits.
Citrate, Phosphate, Lactate)
Functions
• Essential cofactor of more than 300 enzymes.
CLINICAL APPLICATIONS
• Transcellular ion transport.
• Neuromuscular transmission. 1. HYPERCALCEMIA
• Synthesis of CHO, CHON, and Lipids. • Associated with moans, stones, abdominal groans”
• Release of and response to certain hormones. symptoms include kidney stones, abdominal pain,
and depression. Also, too much calcium can be
CLINICAL APPLICATIONS associated with heart rhythm disturbances.
• Causes: parathyroid tumors, other tumors including
1. HYPERMAGNESEMIA
breast cancer, excess amounts of Vit. A or D, Paget’s
• Caused by renal failure and excess antacids. disease, and kidney failure.
2. HYPOMAGNESEMIA 2. HYPOCALCEMIA
• Caused by gastrointestinal disorders; renal diseases; • Associated with eating disorders or lack of
hyperparathyroidism (hypercalcemia), drugs, DM with parathyroid hormone. Symptoms include weakness,
glycosuria and alcoholism due to dietary deficiency. muscle spasms, and heart rhythm disturbance.

METHODS AND INTERFERENCES METHODS AND INTERFERENCES


• Methods used to measure total serum magnesium: • Spectrophotometric – (Ortho-cresolphthalein
Calmagite, methylthymol blue, AAS. complexone, arsenazo III dye), metallochromic
• Measure free (Ionized) Serum magnesium: ISE indicators that bind calcium causing a color change.
• Sources of error: hemolysis, cannot use oxalate, • ISE – specimen must be acidified to convert protein-
citrate, or EDTA anticoagulants. bound and complexed calcium to the free form in
order to measure total calcium.
• AAS (Reference mtd)
F. CALCIUM (Ca) • Measure free ionized serum calcium: ISE,
• The bones are a dynamic store of the calcium in the measurement is temperature sensitive, and generally
body. They are constantly under the influence of the analysis is performed at 37'c.
hormone calcitonin, which promotes bone growth and • Sources of error: Cannot use oxalate, citrate, or
decreases calcium levels in the blood, and EDTA anticoagulants.
parathyroid hormone, which does the opposite. • Interferences: Spectrophotometric- includes
• Calcium is bound to the proteins in the bloodstream, hemolysis, icterus, and lipemia.
so the level of calcium is related to the patient’s
nutrition as well as the calcium intake in the diet.
FEIR, K. ♡
• ISE – includes protein buildup on electrode and • When aminonaphthol sulfonic acid is used to reduce
change in blood pH in vitro before analysis. the complex-colored product is formed and read at
600-700nm.
o Phosphomolybdenum + electrons →
G. PHOSPHATE molybdenum blue
• Found everywhere in living cells, phosphate o Reference range: adults- 2.5-4.5 mg/dL
compounds participate in many of the most important
biochemical processes.
• Phosphate deficiency can lead to ATP depletion, SUMMARY
which is ultimately responsible for many of the clinical • Elevated osmolality is detected by the
symptoms observed in hypophosphatemia. Hypothalamus gland, thirst sensation and
• Phosphate in blood may be absorbed in the intestine secretion of ADH by Posterior Pituitary Gland. ADH
from dietary sources, released from cells into blood, increases renal reabsorption of water.
and lost from bone. • Decrease blood volume stimulates Renin-
Angiotensin-Aldosterone system.
REGULATION • Sodium is the main extracellular cation and
• Phosphate in the blood absorbed from dietary contributor to plasma osmolality.
sources, released from bone. • The renal threshold of sodium is 135-148 mmol/L.
o Regulation occurs by reabsorption/ excretion • Potassium is the main intracellular cation, and the
by the kidneys. electrolytes are mostly affected by hemolysis.
• Most important regulatory hormone is PTH, which • When measuring K+ with an ion-selective
increases renal excretion of phosphate. electrode by means of a liquid ion-exchange
o Vitamin D regulates phosphate by causing membrane, the antibiotic will be incorporated into
intestinal absorption and renal reabsorption. the membrane is valinomycin.
• Chloride is usually a passive follower of Sodium to
DISTRIBUTION maintain electrical charge.
• Although the concentration of all phosphate • In order to maintain electrical neutrality in the red
compounds in blood is about 12 mg/dL (3.9 mmol/L), blood cell, carbonate leaves the red blood cell and
most of that is organic phosphate and only about 3 to enters the plasma through an exchange
4 mg/dL is inorganic phosphate. mechanism with chloride.
• It is predominant intracellular anion, with intracellular • Sodium and Potassium usually move opposite
concentrations varying, depending on the type of cell. each other.
• About 80% of the total body pool of phosphate is • Acidosis is associated with Increase Potassium.
contained in bone, 20% in soft tissues, and less than • Parathyroid Hormone secretion Increases plasma
1% is active in serum/ plasma. calcium, increases plasma magnesium, and
decreases phosphate.
CLINICAL APPLICATIONS • Calcium must be chemically eliminated so that it
will not interfere with the measurement of serum
1. HYPERPHOSPHATEMIA
magnesium.
• Caused by renal failure, hypoparathyroidism,
• Most electrolytes are measured by ISE.
neoplastic diseases, lymphoblastic leukemia, and
intense exercise.

2. HYPOPHOSPHATEMIA
• Caused by diabetic ketoacidosis,
hyperparathyroidism, asthma, alcoholism, and
malabsorption syndrome.

METHODS & INTERFERENCES


• Ammonium molybdate + Phosphate ions →
Phosphomolybdate complex (colorless) read at 340
nm.

FEIR, K. ♡
MT301B – Clinical Chemistry 2 (Lecture) A. ACIDOSIS & ALKALOSIS

April X, 2023 – Acid Base Balance Acidosis


• It develops whenever there is a net increase in the
amount of acid relative to base.
Specimen Collection • Respiratory acidosis results from decreased carbon
• Arterial blood is collected by glass or plastic syringe dioxide excretion.
with glass stopper.
• Arteriolized capillary blood may be used.
• Lyophilized or liquid heparin is the preferred DISTURBANCES IN THE ACID-BASE BALANCE
anticoagulant. Acidosis • pH ↓
Respiratory (lungs)
• 1’ cause: pH↓ PCO2↑ pO2↓
1. Arterial blood collected in a heparinized syringe but (hypoventilation)
exposed to room air would cause which of the
following changes in the specimen? pO2 – elevated; • Metabolic Acidosis
pCO2 – decreased; pH – elevated o Metabolic (kidney)
• No air bubbles should exist because they lower the o 1’ cause: pH↓ HCO3↓
pCO2 values. Blood clots are not acceptable. (bicarbonate excretion)
o Metabolic acidosis results in
• Specimens should be placed on top of ice and must
be analyzed within 15-20 mins. at 4’C and tested increase production of
immediately. Otherwise, pH values decrease and organic acids, ingestion of
acid or acid precursors,
pCO2 values increase.
decreased fixed acid
2. An arterial blood specimen submitted for blood gas
excretion or loss of
analysis was obtained at 8:30 AM but was not
received in the laboratory until 11:00 AM. The bicarbonate.
technologist should: Request a new arterial Alkalosis • pH↑
specimen to be obtained. Respiratory (lungs)
3. The normal range for the pH of arterial blood • 1’ cause: pH↑ PCO2↓ pO2↑
measured at 37C is: 7.35-7.45 (hyperventilation)
4. What is/are the organ responsible for maintaining • Respiratory alkalosis results from
acid-base balance? Lungs and Kidney increased carbon dioxide excretion.
• Lungs (inhalation and exhalation)
o Carbon dioxide (CO2) • Metabolic Alkalosis
o Oxygen (O2) o Metabolic (kidney)
• Kidney (excretion and reabsorption) o 1’ cause: pH↑ HCO3↑
o Bicarbonate (HCO3) (bicarbonate reabsorption)
o Metabolic alkalosis results
Substances that maintain acid-base balance from decreased base
• Buffer system – substances that resist changes in pH. excretion increased acid
o HCO3 – H2CO3 excretion or ingestion of
▪ Major buffer system: ratio is 20:1 base.
o HPO4 – H2PO4
▪ Ratio is 4:1
o Hgb – oxyhemoglobin (HbO2) and 5. A person suspected of having metabolic alkalosis
carbaminoHgb (HbCO2) would have which of the following laboratory findings?
o Plasma protein CO2 content and pH elevated

METHODS
• pH, pCO2, pO2 = directly measured
• pH = glass electrode
• pCO2 = severing Haus electrode
• pO2= Clark electrode
FEIR, K. ♡
• Continuous monitoring • Clinical case: patients with lactic acidosis.
• Transcutaneous electrode • A 1 mmol/L increase in anion gap should cause 1
• Oximeters mmol/L decrease in bicarbonate. Comparing
magnitude of Change in each allows the detection of
B. ACIDEMIA & ALKALEMIA mixed acid base disorders.
• Term refers to alteration in blood pH. • Avoid exposure of specimen to air because loss
• In simple acid base disorders, individuals with carbon dioxide can cause falsely increased anion
acidosis will be acidemia and those with alkalosis will gap.
be alkalemia until compensation is complete.
3. PLASMA ELECTROLYTES
• In mixed acid-base disorders however, blood pH is DETERMINATION
not an accurate determination of acid-base status.
• Plasma electrolytes determination is not central to
• Determination of bicarbonate, blood pCO2, serum evaluation of acid base disorders but may provide
chloride and anion gap give more reliable result in clues to the cause of specific types of acid-base
mixed acid-base disorders. disorders.
• Hyperkalemia is usually seen in metabolic acidosis.
• Hypokalemia is present in almost all cases of patients
A. BASIC SCREENING TESTS (BASIC with metabolic alkalosis.
METABOLIC PANEL)

1. BICARBONATE DETERMINATION B. BLOOD GASES


• Not measured directly, rather, the total amount of • Blood gas measurements are critical to assessment
dissolved carbon dioxide containing compounds is of the respiratory component of the acid-base system.
measured as total Carbon Dioxide.
• Specimen used for analysis is generally arterial blood
• Plasma is alkalinized to convert all dissolved carbon because it provides measure of respiratory function
dioxide to carbonate. and is not being affected by tissue metabolism.
• Carbonate is acidified to liberate Carbon Dioxide. • Heparin is the ideal anticoagulant for arterial blood
• Carbon dioxide can be measured by manometric gas measurement.
method or by titration on automated machines. • Following are the reasons why venous blood is not
• Total carbon dioxide generally overestimates plasma preferred over arterial blood:
bicarbonate by 1 to 2 mmol/L. Because even though o Venous blood has low oxygen content.
total carbon dioxide primarily measures bicarbonate o It has increased hydrogen ion and pCO2.
dissolved carbon dioxide, it contributes about 5% of o Venous pH averages 0.2-0.3 lower than
the value and a small fraction of carbon dioxide is arterial blood.
attached to proteins o Venous pCO2 is 3-4 mmHg higher than
• Current bicarbonate measurement employs arterial blood.
enzymatic method and ion selective electrode. o Decreased perfusion causes greater
6. What is the normal anion gap value? 12-18 mEq/L differences between arterial and venous
2. ANION GAP CALCULATION blood.
• Anion gap calculation is critical in acid-base
disorders.
• Anion gap calculation is the difference between C. MISCELLANEOUS TEST
unmeasured anions and unmeasured cations. • Plasma lactate is measured by enzymatic assays.
• The normal reference range for the anion gap is 6 to • Ketones are often measured by nitroprusside
18 mmol or mEq/L. reaction.
• It is useful in determining increases in unmeasured o Ketone bodies are formed after fatty acid
anions and is calculated as follows: metabolism as source of energy.
o Anion gap = Na – (Cl + HCO3) o B-hydroxybutyrate is usually present at start
(Na + K) – (Cl + HCO3) of ketoacidosis making up to 95% of total
• Increased anion gap occurs when another (usually ketone bodies in urine.
organic) acid is present, and the anion of the acid o With treatment of ketoacidosis the titer of
replaces bicarbonate. ketones in urine rises due to conversion of B-
hydroxybutyrate to acetoacetate.
FEIR, K. ♡
• 3 ketone bodies are B-hydroxybutyrate, acetoacetate, • NaHSO3 – to absorb Oxygen gas.
and acetone.
7. Which among the ketone bodies have the highest B. Automated Blood Gas Analyzers
elevation in blood? ---- • They measure pH, PO2, pCO2.
8. Which among the ketone bodies has the lowest o pH is measured by pH meter or by a
concentration in the blood? Acetone potentiometer
o pO2 is measured amperometrically
♡♡♡♡♡♡♡♡♡♡♡♡♡♡♡♡♡♡♡♡♡♡♡♡♡♡♡♡♡♡♡♡♡♡♡♡
o pCO2 is measured potentiometrically
URINE ELECTROLYTES o HCO3 is measured by calculation using
• Urine electrolytes are helpful in the determination of Henderson-Hasselbalch equation
the cause of non-anion gap metabolic acidosis and o Oxygen saturation is the percentage of total
metabolic alkalosis. hemoglobin that has bound to oxygen and is
calculated from pO2 and pH measurements.
• Urine anion gap is measured using the following
equation: CASE STUDY:
o Urine anion gap = (Na + K) – Cl • A 23-year-old woman was admitted with a 3-day
• Urine anion gap serves as indirect estimate of relative history of fever, cough productive of blood-tinged
urinary excretion of bicarbonate and ammonium ion. sputum, confusion, and orthostasis. Past medical
• Diagnostic significance of urine anion gap history included type 1 diabetes mellitus (DM). A
determination: physical examination in the emergency department
o Strongly negative urine anion gap indicates indicated postural hypotension, tachycardia, and
increased urine ammonium excretion which Kussmaul respiration. The breath was noted to
is an appropriate response to metabolic smell of “acetone.” Examination of the thorax
acidosis. indicated consolidation in the right lower lobe.
o Zero or positive urine anion gap indicates
inappropriate bicarbonate loss in individuals
with metabolic acidosis.
• Urine chloride is also helpful in evaluation of
metabolic alkalosis.
• Changes in chloride concentration indicates the
following:
o Low urine chloride is found in most causes of
metabolic alkalosis.
o Increased urine chloride (> 20 mmol/L)
indicates hyperaldosteronism or Cushing’s
Syndrome.
9. What is the clinical significance of low level of
Aldosterone? Addison’s disease
10. What is the term implied for hypercapnia? Elevated
carbon dioxide

• APPROACH TO DIAGNOSIS
BLOOD GAS ANALYZERS
o The diagnosis of the acid-base disorder
should proceed in a stepwise fashion as
A. Manual Gasometers emphasized in the text.
• These include: o The normal anion gap (AG) is 8–12 (10
o Van slyke Macrogasometer meq/L is a reasonable approximation for
o Natelson Microgasometer calculation purposes), but in this case, the
• The following reagents are used: AG is clearly elevated (20 meq/L).
o Mercury – to create vacuum Therefore, the change in AG (ΔAG) = 10
• Caprylic alcohol – to avoid bubbling of blood. meq/L.
• Lactic acid or sulfuric acid – to release blood gas. o Compare the ΔAG and the Δ[HCO3−]. In
• NaOH – to absorb Carbon dioxide gas. this case, the ΔAG is 10 and the Δ[HCO3−]
FEIR, K. ♡
(25–14) is 11. Therefore, the increment in • These markers may be present in the blood, in other
the AG is approximately equal to the fluids, on cells, or within cells.
decrement in bicarbonate (ΔAG = • Clinically relevant (diagnostically accessible) tumor
Δ[HCO3−]). markers must have high disease sensitivity and
o Estimate the respiratory compensatory specificity, and levels should reflect the status of the
response. In this case, the predicted disease process.
PaCO2 for the patient’s [HCO3−] of 14, • These analytes are typically examined by some form
should be ~29 mmHg. This value is of immunoassay.
obtained by adding 15 to the measured
[HCO3−] (15 + 14 = 29) or by calculating
A. ONCOFETAL ANTIGENS
the predicted PaCO2 from the Winter
equation: 1.5 × [HCO3−] + 8 (±3). In either • Exist as normal proteins in the embryo and fetus and
case, the predicted value for PaCO2 of 29 are also found in certain tumors.
is significantly higher than the measured
value of 24. Demonstrating that the 1. CARCINOEMBRYONIC ANTIGEN (CEA)
prevailing PaCO2 is lower than predicted • An oncofetal antigen used to assess tumors of the
for compensation alone and indicates the colon. It is not as specific as once thought, and not all
coexistence of a superimposed respiratory colon cancers have elevated CEA levels.
alkalosis.
2. α-FETOPROTEIN (AFP)
• In summary, this patient has a mixed acid-base • An oncofetal antigen used to determine the presence
disturbance of two components: (a) high AG of hepatic tumors (hepatoma) and testicular tumors.
acidosis secondary to ketoacidosis and (b)
respiratory alkalosis (which was secondary to 3. CARBOHYDRATE- associated antigen
community-acquired pneumonia in this case). (CA-125)
• The latter resulted in an additional component of • A glycoprotein oncofetal antigen that appears in the
hyperventilation that exceeded the compensatory serum of patients with ovarian cancer.
response driven by metabolic acidosis alone,
explaining the normal pH, and emphasizes the 4. CA-19
concept that physiologic compensation does not • Another glycoprotein oncofetal antigen associated
return the pH to normal. with gastrointestinal tumors.
• The finding of respiratory alkalosis in the setting of
a high-gap acidosis indicates another cause of the 5. PROSTATE-SPECIFIC ANTIGEN (PSA)
respiratory component. Respiratory alkalosis • An oncofetal antigen that is important in screening
frequently accompanies community-acquired and monitoring patients with prostate carcinoma.
pneumonia.
• Taken together, the clinical features of this case 6. BLADDER TUMOR ASSOCIATED
include hyperglycemia, hypovolemia, ketoacidosis, ANTIGENS
central nervous system (CNS) signs of confusion, • Bladder tumor associated antigens include fibrin
and superimposed pneumonia. This clinical degradation products and complement-related
scenario is consistent with diabetic ketoacidosis proteins.
(DKA) developing in a patient with known type 1
DM, when an infection is acquired. Infections are
common precipitating factors in the development of B. PLACENTAL PROTEINS
ketoacidosis in patients with DM. • Synthesized by placental trophoblasts and certain
tumors of the trophoblast.

1. CHORIONIC GONADOTROPIN (CG)


• Used to determine pregnancy, however, high levels
TUMOR MARKERS can indicate tumors of the tests or ovaries, as well as
trophoblastic neoplasia.
• Substances that are synthesized and released by
cancer cells or made by other tissues in response to
the presence of cancer cells. 2. HUMAN PLACENTAL LACTOGEN (hPL)
FEIR, K. ♡
• Used to monitor fetal well-being, but its levels also
can be elevated in patients with trophoblastic
neoplasms.

C. CERTAIN ENZYMES ELEVATIONS


• Certain enzymes elevations can indicate the
presence of cancer cells such as alkaline
phosphatase for bone cancer.

D. INCREASED LEVELS OF SOME


HORMONES
• Increased levels of some hormones indicate
malignancy of the organ that produces the hormone.
• For example, extremely elevated growth hormone
levels might indicate a pituitary tumor.

• DNA analysis determines the proliferative capacity


of tumor cells including those involved in leukemia.
• Molecular diagnosis is used to identify oncogenes,
the genes that transform normal cells into
malignant cells. This type of testing is becoming
more widespread with the advent of simplified
molecular techniques

FEIR, K. ♡
MT301B – Clinical Chemistry 2 (Lecture)

April X, 2023 – Lab Activity

FEIR, K. ♡
MT301B – Clinical Chemistry 2 (Lecture) CLASSIFICATION OF HORMONES
Classification by tissue of origin – releasing factors
Endocrine system → produces hormone 1.
May -, 2023 – Endocrinology but hormone is directly nilalabas sa blood
that act on another tissue to release certain
stream (blood circulation), kaya nahhigh
hormones.
blood kapag may mood swings.
“logy” means system
2. Classification by Structure
ENDOCRINE SYSTEM o Chemical types of hormones: PSA
partner is
CNS
• Manifestations of disease in endocrine glands that ▪ Protein – polypeptides and glycoproteins
may interact of many body systems. In endocrine system → discuss ▪ Steroids
chemicals that will be
• Hormones (product of endocrine system) produced by endocrine gland. ▪ Amines
o Chemical signals produced by specialized
cells, secreted into the blood stream, and GHRH, CRH, TRH, GnRH, Somatostatin,
Proteins/
PRF, ADH, GH, ACTH, PRL, Calcitonin,
carried to a target tissue. Polypeptides
Exocrine – PTH, Insulin, Glucagon
o Ductless (walang path/canal na dinadaanan; diretso
with duct Glycoproteins TSH, LH, FSH, HCG, Erythropoietin
lang sa blood)
Cortisol, Aldosterone, Estrogen,
o Paracrine mechanism – chemical Steroids
Progesterone, Testosterone, Vitamin D
communication between cell within a tissue
Amino Acid Melatonin, Serotonin, Thyroid hormones,
or organ. (Hormone is within the organ/tissue Derivatives Epinephrine, Norepinephrine
“isolated”)
• CNS act in conjunction to maintain the internal
• GHRH, CRH, TRH, GnRH, PRF – mostly present and galing kay
Mother – menopause
Father – andropause

chemical conditions necessary for cellular function


hypothalamus.
(imp. for organ) and emergency demands (mood swings).
• ADH, GH, ACTH, PRL, Calcitonin, PTH, insulin, glucagon –
polypeptide classification
PARACRINE MECHANISM
• Glycoproteins – alpha and beta
Autocrine Signal A chemical that acts on the same cell.
o HCG – alpha and beta (pregnant)
Intracrine Signal A chemical that acts within the same o Erythropoietin (EPO) – produced by kidney; helps bone
cell. (katabing cell na kaparehas niya) marrow to produce RBC, but ‘pag nasa low level of O2;
Neuroendocrine A classical hormone that can be released by kidney ‘pag low O2 level in tissue “hypoxia”.
Signal secreted by specialized nerve cells • Steroid – hormone that needs precursor/partner (cholesterol).
called neurosecretory cells and • Epinephrine & Norepinephrine – found in adrenal medulla.
released into the blood.
1. PROTEIN
• Hormones may be either peptides or glycoproteins.
Classification of the Endocrine System
• They are prohormone which cleaved to produce the
• Cyclicity – product occurs in certain cycle. usual circulating form.
Elevated at 8 AM o Ex: Cortisol – produced by adrenal cortex • The peptide hormones are water-soluble and do not
Decreases at 4 PM (diurnal variation – AM ↑, PM ↓) require transport proteins to move through the blood.
• Pulsatility – increases in a certain time. • They are synthesized and stored within the cell and
o Ex: LH → Pituitary gland, every 55 minutes ↑ have a short half-life (5-60 minutes) in the circulation.

Feedback Mechanism 2. STEROIDS


• HPTA (Hypothalamus Pituitary Gland & Thyroid • Lipid molecules that have cholesterol as a common
Gland Axis) precursor. Water insoluble since may kasama na cholesterol
o Responsible for production of thyroid • It is produced by 4 organs: adrenal glands, ovaries,
hormones. testes, and placenta and have a long half-life (60-100
▪ T3 & T4 – metabolic hormone that targets all minutes) in the circulation.
cell of our body. • Water insoluble and circulate bound to a carrier
▪ Calcium
protein to travel through the blood.
POSITIVE NEGATIVE • Involve in the regulation of sexual development and
When hormone increase When hormone increase characteristics.
eventually leads to increase eventually leads to decrease
of another hormone. (Directly of another hormone and vice 3. AMINES
proportional) versa. (Indirectly proportional) • Derived from amino acids.
Most hormones exhibit this
• Amino acid derivatives (amines) have properties
FM. (for neutralization/balance)
similar to those of both steroid and peptide hormones.
Masakit sa girls; it elevates ‘pag may urine contraction/labor; helps
para mailabas/eject ang milk for breastfeeding. In male, sperm
movement “erection”
4. FATTY ACID DERIVATIVES Oxytocin – responsible for
Oxytocin is also known as
• Includes prostaglandins. uterine contraction during
“love hormone”.
labor. Responsible for
ejection of milk during
INTEGRAL COMPONENTS OF THE ENDOCRINE lactation/breastfeeding.
SYSTEM
• Hypothalamus Parts involved Hyperactivity (↑) Hypoactivity (↓)
• Pituitary Gland (Anterior & Posterior) Anterior Pituitary • Gigantism • Dwarfism
o Anterior – responsible for hormone • Acromegaly • Acromicria
production. (mas malaki; has 6 hormones) • Acromegalic • Simmond’s
o Posterior – does not synthesize any known gigantism disease
hormone but serves as a storage area for • Cushing’s
certain hormones produced by the disease
hypothalamus. (storage of 2 important hormones: Posterior Pituitary Syndrome of Diabetes Insipidus
oxytocin and anti-diuretic hormone) inappropriate
hypersecretion of
AHD (SIADH)
Anterior & ---- Dystrophia
Posterior Pituitary adiposogenitalis
Disorders of Pituitary Gland

2. HYPOTHALAMUS
• Portion of the brain located in the walls and floor of
the 3rd ventricle.
• It’s directly above the pituitary gland and is connected
1. PITUITARY GLAND to the posterior pituitary by pituitary stalk.
• Pea-shaped. • Controls the secretion of hormones from posterior
• Located in a small cavity in the sphenoid bone of the and anterior lobes of the pituitary by different
skull called sell turcica. mechanisms (bec. it controls hormone that is for releasing and
• Anterior pituitary hormones: (Adenohypophysis) inhibiting).

Hormones Target Hormones Target


CRH → ACTH Adrenal cortex → to produce Cortisol Thyrotropin Releasing Pituitary gland → targets thyroid
Hormone (TRH) gland
TRH → TSH Thyroid gland → to produce T3 & T4
Gonadotropin: FSH & Gonads → to produce testosterone Corticotropin Releasing ACTH → targets PG to release
Factor (CRF) ACTH
LH (male); estrogen and progesterone
(female) TitePepe Gonadotropin Releasing Gonadotropin FSH, LH →
PRF → Prolactin Mammary gland → milk Hormone (GRH) targets PG
GHRH → GH All cells (whole body) *whole body Growth-Hormone- GH → targets PG
dahil hindi naman pwede na isang part Releasing Factor (GH-RF)
lang ang lumaki Prolactin Releasing Factor Prolactin
• If may milk and hindi pa pregnant → abnormality (PRF)
Aldosterone hormone – same action (directly prop.) with ADH.
POSTERIOR PITUITARY GLAND (Neurohypophysis) –
Do antagonistic action of
don’t produce hormone.
• Inhibiting Hormones: releasing hormone.
ADH (Vasopressin) –
Found in kidney “RAAS”; i-rerelease o Luteinizing Hormone-Releasing Hormone
promotes reabsorption of lang ‘pag need lang ni kidney para ma-
water.
(LH-RH)
retain ang normal RAAS
o Prolactin Release-Inhibiting Factor (PIF)
ADH is concentrated on kidney • ↑ ADH (blood) → ↑
(RAAS) during water
Urine output (Oliguria) o Somatostatin (Somatotropin-Release
reabsorption; ADH is more on Inhibiting Factor – SRIF)
water reabsorption, nireretain • ↓ ADH (blood
nya ang water sa loob ng body dehydrated) → ↑ Kidney o Oxytocin (Stored here)
by means of retaining it sa blood (Polyuria) o Vasopressin (ADH; stored here)
“Edema” (absorb absorb lang na If decrease, dehydrated because lahat
water and hindi nya nirerelease ay ilalabas sa urine. And pag
sa kidney; therefore, walang masyadong marami ang nilalabas
ilalabas na tubig si kidney. “polyuria”

Aldosterone – hormone that have the same action (directly proportional)


with ADH. Aldosterone hormone that regulates sodium reabsorption; kung
nasaan si sodium andoon rin si water.
ANTERIOR PITUITARY HORMONE
1. GROWTH HORMONE (GH)
• Also known as somatotropin for its growth promoting
effects on the body.
• Most abundant hormone of the anterior pituitary.
• A hormone produced in the liver that stimulates
growth of bone and muscle or linear skeletal growth.
• Complete absence of GH slows the rate of growth to
only one third to one half of normal. (As we age to 22,
naggrow pa rin but slow na lang.)

Hyposecretion Hypersecretion
• Leads to short stature • Leads to acromegaly
but proportionally built. and gigantism (age
• Dwarfism (3 feet) dependent).
• Adolescence –
acromegaly (distal parts:
hands, feet, adolescence)
• Childhood stage –
gigantism
Acromegaly
• Progressive enlargement of the distal parts of the
extremities.
2. PROLACTIN
• A coarse thickening of the bone of the skull, hands, and
feet over the course of the years to decades. • Main target organ is the adult female mammary gland.
Protein synthesis – amino acid buildup for muscles to have • During pregnancy, it helps promote glandular
growth.; Lipolytic – fat cells that can also contribute to growth. enlargement; after delivery it initiates & maintains
• Promoting protein synthesis, lipolytic. st
lactation.1 sign: Breast pain; Glandular enlargement persist at 1
st

• Stimulating fat breakdown. trimester.


• Stimulated by breast feeding.
• Lactogenic
• Males – Prl play a role in growth & development of
• Diabetogenic by increasing resistance to
Function the prostate (kaya may milky portion sa seminal fluid).
insulin (It may elevate blood sugar because
hindi niya pinapakinggan action ni insulin) • This may be because glands that produce hormones
• Stimulates the secretion of recover more quickly than the liver, which absorbs
somatomedins (another hormone produced hormones, leading to high hormone levels. (If may
in liver) problem sa liver, liver will absorb hormones leading to high
• Bioassay hormone level)
o Culture of GH – sensitive rat • Spontaneous production of milk not associated with
Tests lymphoma cells. childbirth, known as galactorrhea, can occur in
o Specimen – serum males and females.
• RIA & IRMA – standard method o April (2018) more recently a report in the
measuring serum GH levels. Canadian Medical Association Journal
• Sleep may be the strongest physiologic
documented a case of a man who had a
Regulation stimulus for GH secretion.
pituitary tumor that result in, among other
• ↑ GH – normally at birth, generally
things, overproduction of prolactin, causing
declines with age.
him to lactate. Amenorrhea – absence of
• Measured in terms of total ug menstruation → another pregnancy.
secreted/24 hrs. • Hypersecretion causes galactorrhea or
• Blood must be collected every 20-30 lactation and is associated with
mins.; the serum immediately separated amenorrhea in women and infertility or
Specimen Pathologic
and frozen until the time of assay. impotence in men. It usually is induced
Conditions
• Urinary GH levels – useful clinically as by pituitary adenoma.
a simple screening tests for acromegaly. • Hyposecretion leads to the lack of
• Measured total renal CHON secretion. lactation in post-partum women.
• Fasting morning blood – for evaluating
Specimen hyperprolactinemia.
• PrL determination levels in urine &
amniotic fluid.
• RIA 4.2 Mineral Corticosteroids
• Chemiluminescence immunoassay • Synthesized from cholesterol, transported bound to
o Coefficient results – 0.81 PrL; CBP and albumin, and are catabolized by the liver
0.92 & 0.98 other hormones and the kidney.
• ACTH – adrenal glands (target)
Assay • The primary action is regulation of electrolytes.
o One adrenal gland is situated
Techniques
on top of each kidney. Each
a. Aldosterone – primary mineralocorticoid
adrenal gland is composed of produced and secreted by the adrenal cortex.
two distinct layers; the adrenal o Functions:
cortex (the outer most region) Once na may defect sa ▪ Stimulating sodium resorption in the
and the adrenal medulla (the mineral corticosteroid, distal convoluted tubules (kidney) in
innermost region) mostly affected result is
exchange for potassium or
electrolyte din. Because
it prod. Aldosterone hydrogen.
GFR which primary action is ▪ Increasing blood volume (via
1. The adrenal cortex is composed of 3 distinct tissues, sodium resorption.
renin/angiotensin system) and
the zona glomerulosa, the zona fasciculata, and
pressure.
the zona reticularis (Sex hormones).
▪ Regulating extracellular fluid volume.
2. The adrenal medulla is composed of chromaffin
• Target tissues of mineralocorticoid action include the
cells.
distal renal tubules and the large intestine.
3. Adrenal hormones
a. Glucocorticoids – steroid hormones produced in • Regulation of aldosterone secretion via the
renin/angiotensin system is achieved as follows:
the zona fasciculata and reticularis of the adrenal
cortex. (Made up of sugar: ↑ Glucocorticoids = Diabetes RAAS: o Decreased blood volume or blood pressure
Mellitus) Renin- induces the release of kidney renin, which
Aldosterone induces the production of angiotensin I and II.
b. Mineralocorticosteroids – steroid hormone Angiotensin
produced in the zona glomerulosa. (Made up of salt) System o Angiotensin II affects release of aldosterone
c. Catecholamines – amine hormones produced in from the adrenal gland, which ultimately
in the adrenal medulla. causes the kidney distal tubule to retain
4. Adrenal endocrine function includes regulation of sodium, thereby raising blood volume and
protein (because of catecholamines – amine), carbohydrates blood pressure.
(because of glucocorticoids – prod. cortisol), and many other
metabolic functions (sex hormone development). 4.3 Catecholamine “Fight-Flight Hormone”
Examine also by urine kasi they
• Catecholamine production is not limited to the adrenal
4.1 Glucocorticoids are excreted by kidney. medulla.
• Synthesized from cholesterol. They are transported • Norepinephrine is predominantly synthesized in the
bound to plasma proteins albumin and corticosteroid CNS, whereas epinephrine is predominantly
binding protein (CBP) are catabolized by the liver and synthesized by the adrenal medulla.
kidneys and are regulated by hypothalamus-pituitary- • Catecholamines are products of the hydroxylation of
Liver is imp. for metabolism and regulates
adrenal axis. hormone. the amino acid tyrosine. They are transported free in
• The primary action of the glucocorticoids is catabolic the blood and regulated by feedback inhibition of
(i.e., promoting protein and lipid breakdown, inhibiting synthesis. Kaya ‘pag galit/stress, catecholamine
protein synthesis). • Functions: is increased.
a. Cortisol – primary glucocorticoid produced and o Mobilization of energy stores by increasing
secreted by the adrenal cortex. Its functions blood pressure, heart rate, blood sugar level
include carbohydrate, lipid, and protein (by stimulating glycogenolysis).
metabolism; suppression of inflammation; o Neurotransmitter actions.
stimulating gluconeogenesis; increasing urine o Release in response to pain and emotional
production (kaya ‘yong mga may hypercortisolism ay ihi disturbance (stress) to mobilize organs.
nang ihi); and stimulating erythropoiesis (prod. of • Tissue targets include the liver and adipose tissue.
RBC). • Approximately 20% of catecholamines are excreted
• Target tissues of glucocorticoid action include the into the urine as metanephrine and
kidney glomerulus, and renal tubules bone marrow normetanephrine: approximately 80% are converted
stem cells, hepatocytes, and adipose tissue. to vanillylmandelic acid (VMA) by the enzyme
Kidney glomerulus – kaya affected ang kidney ‘pag may monoamine oxidase (MAO) → tested in serum.
hypercortisolism.
Bone marrow – kasi it also produces RBC.
Hepatocytes – affected liver cells because carbohydrate, lipid, and
protein metabolism occur in liver.
Adipose tissue – for lipid metabolism (end product of liver).
• Diseases associated with the adrenal cortex center Common cause: 21- • Reactive
on hyperfunction (excess production of bioactive hydroxylase deficiency hypothalamus
(enzyme that converts cortisol (large carbo. meal)
molecules) or hypofunction.
and aldosterone production) • Malnutrition,
Cushing’s Syndrome Cushing’s Disease fasting, binge
• Elevated cortisol • Elevated cortisol alcohol drinking.
• Adrenal adenoma • Pituitary tumor – ACTH
• Elevated steroid • Adrenal hyperplasia
hormones (medication) • Surgery
• ACTH production of lung (pituitary/adrenal)
cancer.
• Treat the cause

Hypersecretion Hyposecretion
Referred to as Cushing’s Causes weight loss,
disease, with symptoms weakness, and
including truncal obesity, gastrointestinal problems.
hyperglycemia, hypertension, “toma/oma” – tumor (benign/malignant)
and protein wasting. It is
caused by pituitary adenoma,
• Pheochromacytoma – major disorder of the adrenal
adrenal hyperplasia, or medulla.
excess production by a o A relatively rare, usually benign tumor arising
nonendocrine tumor. in the chromaffin cells of the adrenal medulla
(https://www.youtube.com/w that results in hypersecretion of
atch?v=FR2r30NXzng)
catecholamines.
Hyperaldosteronism – Hypoadrenalism – caused
involves excess aldosterone by adrenal hypofunction or Bilobed therefore it is separated by a ligament called
production with symptoms of insufficiency and can be 4.4 TSH “Isthmus”
hypertension. These are two induced by three conditions: • The thyroid gland is a bilobed endocrine gland
causes: • Pituitary disease located in the lower part of the neck that is composed
• Conn’s syndrome – involving the adrenal of groups of cells called follicles. Ligament: Isthmus
induced by an cortex (“Addison’s
aldosterone-inducing disease”) which is Hypersecretion Hyposecretion
adrenal adenoma or relatively rare → Causes thyrotoxicosis and Difficult to differentiate from
adrenal hyperplasia and hypoglycemia non-diabetic is the result of either primary hypothyroidism.
is a rare cause of • Secondary adrenal hyperactivity of the pituitary
hypertension. (problem: insufficiency gland, leading to increase
cell of adrenal gland) precipitated by TSH release.
• Excess renin production decreased levels of Thyroid gland will not produce TSH if wala/kulang ng
leads to elevated corticotropin releasing iodine.
aldosterone level. (Renin hormone (CRH) or • This gland contains 2 cell types:
increases na galing kay ACTH (hypothalamus may o Follicular cells produce the hormones
kidney) problem kasi does not thyroxine (T4) and triiodothyronine (T3)
secrete/release ng
o Parafollicular cells (lying adjacent to the
CRH/ACTH)
• Long-term suppression
follicles) produce the hormone calcitonin.
of the hypothalamic- A. Thyroid hormones require iodine for their
pituitary-adrenal axis by synthesis.
glucocorticoids. Thyroglobulin (insideo The iodine combines with the protein
the follicles) is rich in
Congenital adrenal Hypoglycemia – non- tyrosine. Kaya papasok
thyroglobulin to form hormones precursors
hyperplasia – a genetic diabetic sakanya si Iodine that in turn combine to form T3 and T4.
disorder causing a deficiency • Endocrine (I + T) through TSH ▪ Thyroglobulin (follicles) – Tyrosine +
of enzymes in the synthetic • Addison’s receptor.
Iodine from TSH receptor. Iodine
2 form that will enter
pathways that lead to cortisol • Renal/liver failure thyroglobulin: Mono & (mono) + Tyrosine →
and aldosterone production. • Insulinoma Diiodine Monoiodothyronine (MIT)
• Medication Iodine 1 + Tyrosine →
ACTH levels are increased, Monoiodothyronine
▪ MIT + DIT → T3 ; DIT + DIT → T4
(Quinine, Propanol)
and steroid hormones are Iodine 2 + Tyrosine → (Metabolic hormone)
hypersecreted. Diiodothyroninea. The hormones are either stored within the follicle
or released into the bloodstream.
o In the blood, most T4 eventually gives up an hands and multinodular goiter
iodine molecule and forms T3. face, course (gland produces excess
b. Approximately 98% of circulating T3 and T4 is skin) occurs as hormones) → need na
the disease operahan
bound to protein, including thyroxine-binding
progresses. • The laboratory
globulin (TBG) and thyroxine-binding albumin.
• Causes of evaluation of
Some hormone remains unbound or free, and this hyperthyroidism in the
hypothyroidism relate to
is the physiologically active fraction. the area of tissue initial evaluation reveals
o Lab. Test: Free T3 & Free T4 “FT3 & FT4” elevated thyroid
damage. In addition,
c.Thyroid hormone function includes action at the hypothyroidism can be hormone serum levels
cellular level to regulate carbohydrate, lipid, and caused by lack of dietary and decreased serum
protein metabolism. The hormones also act on iodine. TSH.
the CNS, stimulate the heart, and have a role in • Primary – involves the • Grave’s disease – an
physical growth and development. inadequate secretion of autoimmune disorder
d. The regulation of T3 and T4 occurs in the thyroid hormones that occurs six times
following manner: caused by a damaged or more frequently in
o Thyroid releasing hormone (TRH) is released surgically removed women than in men. In
thyroid gland. (Thyroid this disorder,
by the brain and stimulates the release of
gland lang ang may defect) immunoglobulins
TSH (thyrotropin) from the pituitary gland. Congenital stimulate the thyroid
o TSH stimulates iodine uptake by the thyroid hypothyroidism is gland by binding to TSH
gland and also causes the release of T3 and caused by the absence receptors (either walang
T4 from the thyroid gland. of the thyroid gland. TSH receptor or
o High serum levels of free T3 and T4 “shut Laboratory results: nagmmalfunction bec. TSH
decreased T3, T4 free receptor is the one that
off” the release of TSH from the pituitary
thyroxine index (FT4I), regulates the entry of
gland, whereas decreased levels induce TSH Iodine to follicular
T3 uptake (T3U), and
release. cells/thyroglobulin cells)
increased TSH.
B. Thyroid disorders are caused by increased or Symptoms: similar to
• Secondary – involves
decreased levels of the circulating hormones T3 and those of
decreased production of
T4. A wide variety of physical diseases can be traced hyperthyroidism.
TSH caused by pituitary
Laboratory result:
back to a dysfunctional thyroid gland. disorder leading to low
increase T3, T4, FT4I,
serum levels of the
Hypothyroidism Hyperthyroidism and T3U and
thyroid hormones.
• A serum level of thyroid • Caused by excessive decreased/normal TSH.
(Pituitary gland ang may
Serological test which is
hormone that is thyroid hormone in the defect)
anti-TSH receptor
insufficient to provide for circulation. This causes Laboratory results: all
• Thyroiditis – an
the metabolic needs of cells to become thyroid test values are
inflammation of the
cells. overactive. The disorder decreased.
thyroid gland caused by
• This disorder affects is sometimes referred to • Tertiary – caused by
either bacterial or viral
women four times more as thyrotoxicosis. hypothalamic failure
infection.
than men between the • Symptoms: leading to a lack of TRH
(https://www.youtube.c
ages of 30 and 60 years. o Weight loss, production.
om/watch?vUrx6C5gm
• Hypothyroidism is loss of muscle (Hypothalamus lang ang
aFA)
usually referred to as mass, may defect)
primary, secondary, or hyperactivity
tertiary, depending on yet quick • Chronic immune
the site of the fatigability, thyroiditis
dysfunction. insomnia, (Hashimoto’s disease) Hashimoto’s Disease –
• Symptoms: increased – caused by a genetic nawawala ang TPO;
o Enlarged sweating, abnormality in the therefore, walang maproduce
thyroid gland nervousness, immune system and na DIT/MIT → lead to
(goiter), fatigue, palpitations, hypothyroidism →
involves massive
goiter, and Magkakaroon ng serological
impairment of infiltration of the thyroid test which is anti-TPO.
mental bulging eyes. gland by lymphocytes.
processes, and • Causes of this disorder The symptoms match
loss of appetite. include pituitary tumors those of hypothyroidism
o Myxedema that cause excessive
(loss of hair, TSH secretion, thyroid Para ma-form si T3 and T4, kailangan mag-combine si Tyrosine +
swelling of the carcinoma, or toxic I2; Tyrosine + I1 → they will form through enzyme Thyroid
Peroxidase (TPO) to convert into DIT.
Hypothyroidism Hyperthyroidism of
1’ TRH elevated 1’ TRH low level endometrium
Hypothalamus 2’ TRH elevated 2’ TRH low level Corpus Progesterone Uterus Maintenance
3’ TRH low level 3’ TRH elevated luteum Mammary of
1’ TSH elevated 1’ TSH low level Gland endometrium
Pituitary Gland 2’ TSH low level 2’ TSH elevated lactation
3’ TSH low level 3’ TSH elevated Testes Androgens Male Growth and
1’ low level T3T4 1’ elevated T3T4 (Testosterone) prostate, development
Thyroid Gland 2’ low level T3T4 2’ elevated T3T4 Genitalia, ; maturation
3’ low level T3T4 3’ elevated T3T4 Larynx,
Skeleton
FSH grows ovum, while LH once ovum already growth
• If primary (1’) hypothyroidism; TG ang pasaway/problem → and mature, LH will trigger it to release sa ovary.
inversely proportional kay hypothalamus and pituitary gland. A. FSH & LH are glycoprotein hormones synthesized in
• If secondary (2’), PG ang may problem → inversely proportional
the anterior pituitary gland and transported unbound
kay hypothalamus.
• If tertiary (3’), hypothalamus ang may problem → susundan ni PG
via the systemic circulation to target tissues.
and TG ang hypothalamus since normal lang sila. a. In a woman, FSH induces the growth of the ovum
inside the follicle, and LH triggers release of the
• (https://www.youtube.com/watch?v=QG-UbtFEc_c) ovum.
(https://www.youtube.com/watch?v=KmPKx-AhYPU) b. In a man, FSH induces spermatogenesis in the
GRH will be released by hypothalamus that targets pituitary gland to Sertoli’s cells of the testes, and LH stimulates the
produce gonadotropins for FSH and LH. production of testosterone by the Leydig’s cells.
4.5 Gonadotropins (FSH and LH) B. Progesterone, testosterone, androgens, estrone,
• (https://www.youtube.com/watch?v=92IUxit7aWs&t= and estradiol are steroid hormones synthesized
556s) either in the testes, ovaries, or adrenal glands. Each
of these steroid hormones has cholesterol as its
Hypersecretion Hyposecretion precursor.
Results in sexual precocity Results in sexual a. The ovaries produce estrogens (i.e., estrone,
and is usually a result of underdevelopment and Lahat para sa
‘pag magme- estradiol), androgens and progesterone.
brain tumors in the region of infertility. female
mens na, uterus
o The estrogens are responsible for
the hypothalamus. lining thickens → it
means that development of the uterus “bahay-bata”,
anytime that ovary fallopian tubes, and female sex
Hormone is open & ovum characteristics.
Endocrine Target waits for sperm
cell, it’s a sign na o Estrogens
Produced/Rele Response also prepare the uterine
Gland Tissue
ased kumakapal ang endometrium (lining) for pregnancy.
Hypothala LH-releasing Anterior LH/FSH lining of matres, o Progesterone prepares the breast for
mus hormone Pituitary secretion waiting and ready
na for pregnancy lactation and maintains the endometrium.
Anterior LH, FSH Ovarian Ovulation
once na-fertilize 1. Estradiol – chief estrogen produced
Pituitary follicle, Progesteron ang egg. Ninipis
Luteal cell, e produced
specifically by the maturing follicle within the
‘pag walang na-
Leydig cell, Testosterone meet na sperm ovary. (puberty until days before menopause)
Ovary produced cell and will 2. Estrone – other major biologically important
proceed to estrogen, is produced either in the peripheral
Sertoli cells Follicular
menstruation.
maturation; tissues via conversion of prohormone
Estradiol androstenedione or in the ovary from the
produced conversion of estradiol. (nag-menopause na/near
Spermatoge menopausal stage)
nesis 3. Another estrogen, estriol – produced in the
Ovary Estrogens Hypothala FSH inhibited placenta. Therefore, little is present in
(follicles) mus, FSH
nonpregnant women. (elevates if pregnant)
Pituitary inhibited; LH
4. Progesterone – produced by the corpus
Gonads, secretion
Uterus Secondary luteum (the structure that forms in a follicle
sex that has released its ovum). It is present in
characteristic significant amounts following ovulation.
develop 5. Androstenedione – chief female androgen.
preparation (nag-aadd ng male characteristics since ito rin ang
mataas na hormone ng male)
b. The testes in adult men produce testosterone and o Abnormalities of testicular function focus on
small amounts of androstenedione, the lack of or excessive production of
dehydroepiandrosterone, and estradiol. androgens.
o Testosterone (the most potent of the o Both increased and decreased androgen
androgens) induces growth of the male production can be a primary condition
reproductive system, prostate gland, and resulting from testicular dysfunction, or it may
development of male sex characteristics, be a secondary condition resulting from
including hypertrophy of the larynx (lumalaki hypothalamus-pituitary dysfunction.
Androgens - voice ng lalaki) and initiation of D.
galing sa
adrenal cortex spermatogenesis.
Hypergonadism Hypogonadism
(reticularis) o The other androgens are also responsible for
• Most often a condition • Symptoms: directly
the secondary sex characteristics in men.
resulting from excessive dependent on the time of
c. Adrenal gland androstenedione is converted to androgen production by the development of
testosterone in the peripheral tissues and a testicular tumor. androgen deficiency.
accounts for approximately 5% of the total • It can also occur • Pre-pubertal
testosterone in men. secondary to hypogonadism –
C. The regulation of all reproductive processes involves hypothalamus-pituitary caused by the absence
interrelated events and endocrine systems, including dysfunction, with a of androgen production.
the hypothalamus, pituitary gland, and the gonads. resulting ↑ in FSH/LH Infantile genitalia
a. In women, the menstrual cycle controls secretion. persist; growth
reproductive events. High serum levels of FSH • In the adult, there is little continues but is
physical change with decreased.
induce follicular development and estrogen
these disorders. This condition is often
release. The increased level of estrogen causes not apparent until the
However, increased
a decrease in the release of FSH at the level of androgen levels in adolescent period when
the pituitary gland and also causes an increase in children result in normal adolescent
serum LH. precocious puberty. development does not
o LH initially causes estrogen levels to decline occur.
Kapag lumabas
then induces maturation and rupture of the • Post-pubertal
na si ovum, ang
follicle, which releases ovum. Estrogen levels hypogonadism –
tawag na sakanya
ay corps luteum. increase following this release, the ruptured results in minimal
Then, si changes.
follicle becomes the corpus luteum, and
progesterone na • Primary – caused by a
ang mapproduce. progesterone is produced. Androgenic feedback –
lack of androgenic
o Progesterone inhibits LH release from the connection of pituitary
feedback by FSH/LH on
gland to
pituitary gland. (hindi pwedeng continuous na androgens/gonads. the hypothalamus-
mataas si LH) pituitary axis, which is
o Serum FSH & LH levels continue to decline, typically caused by a
and regression of the corpus luteum induces genetic defect in
decreased serum levels of estrogen and testicular development.
progesterone, initiating menstruation. Laboratory results:
(magllower na si estrogen and progesterone hanggang ↑ serum and urine
sa mag-menstruate na; if mataas si FSH and LH – mood gonadotropins
swings – sign na malapit na magkaroon) (LH/FSH),
o The estrogen-induced inhibition of FSH ↓ serum androgens
release by the pituitary gland is removed, and (testosterone), and
FSH levels begin to increase, beginning the ↓ urinary 17-
cycle again. ketosteroids.
o Menstrual cycle • Secondary – caused by
(https://www.youtube.com/watch?v=6e- primary hypopituitarism
or a hypothalamic
YIruVSLM)
dysfunction that results
b. In men, increased levels of serum testosterone
in ↓ production of LH and
inhibit the release of LH from the pituitary, thereby FSH by the pituitary
decreasing the production of testosterone by gland.
Leydig’s cells. Laboratory results:
Absence of serum and
urine gonadotropins and primary condition 1. Tumors and necrosis
↓ serum androgens. associated with this of the pituitary gland
disorder. (Sheehan’s
Syndrome)
E. Abnormalities of ovarian function present as either 2. Congenital
hypo- or hyperfunction, both of which are considered hypothalamic
as either primarily caused by ovarian disease or disorders
secondarily caused by hypothalamus and pituitary 3. Illnesses: Congenital
dysfunction. heart disease,
chronic renal
a. Ovarian hypofunction is also a time-dependent
disease, rheumatoid
condition.
arthritis, rapid weight
o If it occurs before the onset of puberty, it loss, anorexia
induces delayed or absent menstruation. nervosa, or
o If it occurs after the onset of puberty, it will hyperthyroidism.
result in secondary amenorrhea (lack of
menstruation).
b. Ovarian hyperfunction is usually caused by
estrogen-secreting tumors or may be idiopathic in
origin

Hyperfunction Hypofunction
• Primary Ovarian • Primary Ovarian
Hyperfunction – Hypofunction – caused
result of the presence by a lack of estrogenic
of an estrogen- feedback on the
secreting tumor, which hypothalamus-pituitary
results in decreased system, which results in
serum FSH/LH levels increased release of FSH
irregular uterine and LH from the pituitary
F. Other conditions involving female reproductive
bleeding is a typical gland.
hormones include:
symptom. • Characterized by
increased serum levels of a. Hirsutism or excess hair along the midline of the
gonadotropins and female body (i.e., lip, chin, chest)
decreased estrogen b. Polycystic ovary syndrome (PCOS) or
levels. There are two enlarged ovaries – associated with infertility and
common causes of this other menstrual irregularities.
condition: c. Infertility – caused by lack of ovulation in women
1. Menopause – and inappropriate sperm production in men.
termination of
reproduction in
womenTurner’s G. Laboratory analysis of reproductive endocrinology
syndrome – a involves serum and urine testing of a variety of
congenital endocrine hormones and their metabolites or function tests that
disorder in which the involve stimulation or suppression of various
ovaries cannot
hormones.
secrete estrogen.
(Monosomy 23)
• Secondary Ovarian • Secondary Ovarian
Hyperfunction – Hypofunction – results
idiopathic and results from hypothalamic-
in increased estrogen pituitary dysfunction or
secretion caused by serious illness.
the presence of Laboratory results:
increased FSH/LH decreased serum
levels. Sexual gonadotropins, estrogen,
precocity (early and progesterone.
development of 3 most common causes
secondary sex of this condition are:
characteristics) is the
Too much water in blood → nababwasan water sa kidney → lack water and
super dami ng nutrients in brain (nalulunod ang brain cells) kaya nagkakaroon Found at the back of
ng meningitis. thyroid gland.

POSTERIOR PITUITARY HORMONE PARATHYROID HORMONE (PTH)


1. ANTIDIURETIC HORMONE (ADH) – • Produced by the parathyroid glands, which are small,
VASOPRESSIN paired structures located in the posterior thyroid
• Regulates water reabsorption (happens in kidney) and capsule.
blood pressure by affecting the renal tubules and the • They may, however, be located in other parts of the
arterioles. neck or upper chest cavity.
• The sole function of the parathyroid gland is the
Hypersecretion Hyposecretion production of parathyroid hormone.
• Results in conditions • Associated with 1. PTH synthesis begins with the precursor pro
including meningitis, Diabetes Insipidus
parathyroid hormone
head injury, (DI).
a. The major physiologic action of PTH is mineral
tuberculosis, • This disorder results
hypoadrenalism (if from destruction of the Complex bec. homeostasis. Specifically, PTH is involved in the
tumaas si ADH, bababa si posterior pituitary gland may + and - metabolism of both calcium and phosphorus by
adrenalism; bec. Kulang feedback the kidney and bone. A complex interrelationship
or the hypothalamus.
water sa kidney kaya
• No known disorders are exists between PTH, cholecalciferol (vitamin D)
affected ang adrenal
associated with excess and calcitonin.
gland), hypothyroidism (if
or deficient secretion of b. Transport. Parathyroid hormone, unlike many
sobra ng water sa blood,
dadadan sa thyroid gland oxytocin. other proteins, is transported as a freely
and malulunid thus hindi circulating, intact, active molecule. The
magpproduce ng maraming
biologically inactive amino acid fragments can be
TSH), and cirrhosis.
detected in the serum as well.
• SIADH – associated
with hyponatremia and
c. The primary determinant of PTH release is the
hypertonic urine, despite serum concentration of ionized calcium,
normal renal and considered to be the biologically active form of
adrenal function. serum calcium. Other substances that impact
• Symptoms: Weakness, • Symptoms: Insatiable PTH secretion rates include:
malaise, poor mental thirst (kasi lagging ▪ Magnesium, which has an effect on PTH
status, convulsion, and nilalabas ang water sa that both parallels and modulates the
coma. It is typically body), polydipsia, and
effect of calcium on PTH release.
caused by release of polyuria.
▪ Biogenic amines like epinephrine,
ADH from ectopic
dopamine, and serotonin.
tumors.
▪ Vitamin D (sunlight and milk)
2. PTH has an important role in calcium and phosphorus
Tissue of Origin Hormone(s) Produced metabolism.
Hypothalamus Thyrotropin-releasing hormone, a. Calcium is a mineral proved to be essential for
Corticotrophin-releasing factor, Other heart muscle contraction, hemostasis, and cell
releasing factors responsiveness.
Anterior Pituitary TSH, ACTH, FSH, LH, Prl, GH ▪ Calcium homeostasis is regulated not
Posterior Vasopressin, Oxytocin
only by PTH but also by cholecalciferol
Pituitary
(vitamin D) and the thyroid hormone
Adrenal Medulla Epinephrine, Norepinephrine
calcitonin.
Adrenal Cortex Cortisol, Aldosterone, 11-deoxycortisol
Thyroid Triiodothyronine, Thyroxine, Calcitonin
▪ The overall effect of PTH is to raise blood
Parathyroids Parathyroid hormone calcium levels through its action on bone
Pancreas Insulin, Glucagon and kidney.
Gastrointestinal Gastrin, Others ▪ In bone, PTH increases bone resorption
Tract of calcium into serum. (siya ang kumukuha ng
Ovaries Estrogen, Progesterone calcium sa bone papunta sa blood →
Placenta Progesterone, HCG, Human placental osteoporosis/weaking of bone)
lactogen ▪ In the kidney, PTH increases the renal
Testes Testosterone, Other androgens reabsorption of calcium. (excss calcium from
Kidney 1,25-(OH)2, Vitamin D, Erythropoietin serm ay dadalhin sa kidney para mag-reabsorb si
kidney ng calcium; ↑PTH in kidney = renal stone)
Unknown Prostaglandins
b. Phosphorus, or rather compounds of phosphate,
Produced
is in all living cells and participates in many
by ATP
important biochemical processes.
▪ The overall effect of PTH is to lower disorders, • Clinical manifestations:
phosphorus concentrations, whereas collectively referred are those of hypocalcemia
Vitamin D acts to increase blood to as multiple and include tetany (muscle
phosphate. endocrine neoplasia spasms), skin changes
(MEN), which involve (especially drying), brittle
c. Calcitonin produced by parafollicular cells in the
the pituitary, hair, Hypotension, and GI
thyroid. pancreas, thyroid, upset.
▪ In general, the overall effect of calcitonin and adrenal glands. • Low serum calcium and
If tumaas si PTH,
kailangan bumaba ni is to decrease blood calcium because of Serum calcium is high phosphorus are
calcitonin. Kasi ‘pag its effect on both bone and renal calcium increased. hallmarks of this disorder.
tumaaas si calictionin, processing, which is just the opposite of • PTH is increased,
iddecrease niya ang
blood calcium → PTH. and phosphorus is
POSITIVE FEEDBACK▪ In bone, calcitonin inhibits bone normal to decreased.
resorption of calcium. • Secondary • Idiopathic
▪ In the kidneys, calcitonin decreases the Hyperparathyroidis Hypoparathyroidism –
m – a condition rare and can be hereditary
renal reabsorption of calcium,
associated with an or seen in conjunction with
phosphorus sodium, potassium, and
attempt for the body other endocrine disorders.
magnesium. to compensate for • A very low serum calcium
3. Pathologic conditions involving a dysfunctional hypocalcemic states. level and a very high
parathyroid gland most often manifest as (Onti calcium kaya phosphorus level are
hypocalcemic or hypercalcemic states. need mag-elevate ni indicative of this condition.
PTH)
a. Hypocalcemia – classified as either being
• It is commonly seen
associated with deficient PTH concentration or
in patients with renal
being independent of PTH activity (vitamin D- failure who cannot
deficient states) → hypo because deficient si PTH, wlang excrete phosphorus,
magrereabsorb ng calcium kay bone papunta kay serum; resulting in a
kaya bababa calcium level ni serum)
decrease in calcium,
▪ In general, the PTH-dependent which stimulates the
hypocalcemic disorders fall into two secretion of PTH.
categories: those related to • Serum calcium is
hyposecretion of PTH and those related low, PTH is
to tissue resistance to PTH (known as increased, and
pseudohypoparathyroidism). phosphorus is
b. Hypercalcemia – diagnosed when serum increased.
calcium levels rise to >102 mg/L or are sustained • Pseudohypoparathyroidi
at levels >100 mg/L. sm (PHP) – characterized
by lack of responsiveness
▪ Symptoms range from no symptoms to
to PTH by the renal system
manifestations of polyuria, polydipsia, or other organ systems, not
kidney stones, acid-base disorders, by a decrease in PTH.
nausea, stupor, or coma. (kidney ang may problem)
▪ Malignancy (e.g., multiple myeloma, • Serum PTH levels are
leukemia, lymphoma) and typically normal to increase
hyperparathyroidism account for most in this condition.
cases of hypercalcemia.

Hyperparathyroidism Hypoparathyroidism Bone (99&) Blood (1%)


• Primary • Primary ↑ PTH → All Ca+ dadalhin sa Hypercalcemia →
Hyperparathyroidis Hypoparathyroidism – blood Hypophosphatemia (all
m – most often synonymous with Phosphorus <------------------- phosphorus pupunta kay bone)
results from hyposecretion of PTH. (inorganic)
parathyroid • It is caused by surgical ↑ Calcitonin (since nagkaroon Hypocalcemia →
adenoma or removal of the parathyroid ng hypercalcemia, all calcium ay Hyperphosphatemia (since
hyperplasia. dadalhin kay bone) maraming pumasok na Ca
glands, trauma following
Phosphorus ------------------->
• Additionally, surgery, or radiotherapy kay bone, need ni
(inorganic) phosphorus mag-exit
hyperparathyroidism directed toward the thyroid
can be associated gland. papunta kay blood
with other endocrine “nagtampo”
↑ Vitamin D (if maraming Hyperphosphatemia
phosphorus, Vit. D will help
phosphorus para mag-
elevate)

YouTube Links:
• Hyperparathyroidism
(https://www.youtube.com/watch?v=sD9st1ZPFrQ;
https://www.youtube.com/watch?v=UmXHhsPwGP0)
• Hypo and Hyperthyroidism
(https://www.youtube.com/watch?v=Urx6C5gmaFA)
• Hormones of Kidneys, Heart, and GIT
(https://www.youtube.com/watch?v=T-GBsFKn5c4)
• Pancreas
(https://www.youtube.com/watch?v=LKEf9bBotM8)
• Physiology of Pancreas
(https://www.youtube.com/watch?v=-3J6QRMerQE)
• How Insulin Works
(https://www.youtube.com/watch?v=HJGjNTJgf48)
• Insulin and Glucagon with CHO Metabolism
(https://www.youtube.com/watch?v=Id2E72P8Fe0
MT301B – Clinical Chemistry 2 (Lecture) Handheld Immunoassays
• Most common type of drug screen.
May -, 2023 – Toxicology
• They detect a wide variety of drugs but cannot
separate closely related compound.
A. ELEMENTS OF TOXICOLOGY • Blood and urine can both be analyzed with this
• Toxicology – study of toxic drugs or poisons. methods.
• Exposure to toxins may be due to suicide attempt,
Gas-liquid Chromatography
accidental exposure, or occupational exposure.
o Toxicants – any substances that when taken • Allows for greater sensitivity in the identification of
in sufficient quantity, causes sickness or drugs.
death. • It can be used as a confirmatory technique for drugs
o Toxicity – relative term used to compare one detected by screen.
substance with another.
▪ Toxic substance – one with a 2. CONFIRMATORY DRUG TEST
toxicity defined as “extremely” or • Required to confirm and quantify those drugs found in
“super” toxic. a patient’s serum or urine using drug-screening
• Routes of exposure methods.
o Ingestion Gas Chromatography/Mass Spectrometry
o Inhalation
• Sensitive technique used to confirm drugs detected
o Transdermal absorption
by screening techniques.
• Toxic Response – the amount of damage done to an
• Typically, urine samples are initially analyzed by gas
organism when the substance is administered at less
chromatography to determine the presence of
than the lethal dose.
compounds, then reanalyzed by mass spectrometry
• Acute Toxicity – a one-time exposure of short
to examine fragments of these compounds for relative
duration to an agent that immediately causes a toxic
abundance in the sample.
response.
• Chronic Toxicity – multiple exposures for extending Immunoassay Techniques
time periods to an agent at a dosage that will not • Uses antibodies to detect drugs.
cause an acute response. • These methods are usually automated and in the form
of enzyme immunoassays.
B. ANALYSIS OF TOXIC AGENTS Ethanol Testing
• Screening Test • Typically performed using gas chromatography.
o Performed first and usually of a qualitative • However, an enzyme assay using alcohol
nature. dehydrogenase and measuring the increase in NADH
o May lack specificity. formation following the reaction is widely used and
• Confirmatory Test can be automated.
o Usually quantitative with good specificity and
sensitivity. Heavy Metal Testing
o Example: Gas Chromatography and • Most often performed by atomic absorption
Immunoassays spectrophotometry.

1. SCREENING TEST
C. ANALYSIS OF SPECIFIC SUBSTANCES
• The drug screen rapidly identifies a drug or drugs
present in the blood, urine, or gastric contents of a 1. ALCOHOLS
patient suffering from toxicity.
• Volatile organic substances.
• Neutral and basic drugs as well as drug metabolites
are best detected in urine, whereas acidic drugs are Ethanol
best found in detectable concentrations in blood and • Chronic exposure is associated with toxic hepatitis
serum. and cirrhosis.
• Following a positive drug screen, confirmatory • Acts by depressing the CNS and increasing the heart
methods must be used to quantitatively analyze drug rate and blood pressure.
levels in patients. • Analysis: Enzymatic & Gas-liquid Chromatography
o Ethanol + NAD ADH acetylaldehyde + NADH
o GLC is the reference method, because it can Lead
differentiate among the various types of • Lead poisoning is most typically caused by lead paint
alcohols and quantify them. ingestion or continuous exposure to lead in soil.
o Lead binds to proteins and it inhibits many
Methanol
enzymes; it also inhibits heme synthesis.
• Ingestion is related to severe acidosis, blindness, and
o Toxicity may occur when lead is ingested,
even death due to methanol conversion to
inhaled, or contacted dermally.
formaldehyde, which is metabolized to formic acid.
• Lead interferes in heme biosynthesis at several
• Widely used in paints, solvents, antifreeze, and solid
stages, the last of these being the incorporation of
canned fuels.
iron into the tetrapyrrole ring.
Isopropanol o This alterations results in the formation and
• Ingestion produces severe, acute symptoms, similar accumulation of zinc protoporphyrin (ZPP),
ethanol intoxication, that persist for a long period of with zinc replacing the iron in the tetrapyrrole
time because isopropanol is metabolized to acetone, ring.
which has a long half-life. • Lead poisoning in children is generally associated
with the ingestion of lead-laden paint chips.
Ethylene glycol (found in anti-freeze) o Laboratory results indicate:
• -- ▪ Basophilic stippling of RBCs.
▪ Increased urinary excretion of
2. CARBON MONOXIDE aminolaevulinic acid and
• A tasteless gas with 200 fold greater affinity for corpoporphyrins.
hemoglobin than oxygen. • Acceptable blood lead level: <10 ug/dL in young
• Toxic because it binds very tightly to hemoglobin and children
does not allow oxygen to attach to the hemoglobin; • Lead analysis:
forms carboxyhemoglobin. o Whole blood specimen required
• Produces hypoxia in brain and heart. o Methods: AAS and Anodic Stripping
• Whole blood is required for analysis. Voltammetry
• Analysis: GLC is the reference method.
Mercury (Mercury Salts)
3. CYANIDE • Found in antibacterial agents, photographic reagents,
pesticides, and batteries.
• Super toxic substance with exposure occurring
through various routes, including oral ingestion, • Poisoning by these agents is typically the result of
inhalation, or transdermal absorption. overexposure or ingestion.
• Used in insecticide and rodenticide products. • Bind to proteins and inhibits many enzymes.
• Cyanide binds to heme iron and mitochondrial • Analysis: AAS and Anodic Stripping Voltammetry
cytochrome oxidase.
• Analysis: ISE 5. PESTICIDES
• Includes insecticides and herbicides that may
4. METALS contaminate food or be inhaled, absorbed through the
skin, and ingested via hand-to-mouth contact.
Arsenic • Exist as organic complexes, organophosphates
• Found in pesticides, weed killer, and some paints. (largest single group of pesticides), and
• Binds thiol groups in proteins; ionized arsenic carbohydrates.
excreted in urine. • These compounds inhibit AChE, which results in
• Specimens: specific effects on the heart and respiratory centers,
o Blood & Urine – used to assess short-term muscle cramps, and certain CNS effects.
exposure. • Analysis: Assess enzyme activity of erythrocyte
o Hair & Fingernails – used to assess long- acetylcholinesterase or serum pseudocholinesterase.
term exposure. • Antidote: The effective antidote for pesticide
• Analysis: AAS poisoning is atropine sulphate.
6. THERAPEUTIC DRUGS COMMONLY Cannabinoids Compounds
ABUSED • Function: Hallucinogenic, provide a feeling of mental
• Analgesics – anti-inflammatory agents and well-being and euphoria, impair mental function and
painkillers. short term memory.
o Produce psychologic effects and are stored
Salicylate (Aspirin) in fat cells.
• Function: Used as an analgesic, antipyretic, and • Marijuana-THC (Tetrahydrocannabinol) – primary
anti-inflammatory. cannabinoid component; THC is excreted in the urine
• Metabolism: Administered orally for an extended period of time depending on use.
• Toxic effects at high dosages: Salicylate (aspirin) is o Overuse of this drug is rare and is not severe
considered toxic at the serum level of >90 mg/dL, 6 enough to be life-threatening.
hrs. following ingestion. o THC half-life in blood is 1 day following single
o Causes mixed acid-bases imbalance seen as use and 3-5 days following chronic use.
metabolic acidosis and respiratory alkalosis, • Metabolism: THC distribute in lipophilic tissue such
ketone body formation, excess formation of as the brain and adipose tissue. Elimination is
lactate. dependent on THC being metabolized by the liver to
• Analysis: Ferric nitrate method with colored product 11-nor-tetrahydrocannabinol-9-carboxylic acid (THC-
read spectrophotometrically; gas or liquid COOH)
chromatography. o With this metabolic product filtered by the
kidney, THC-COOH (major urinary
Acetaminophen (Tylenol) metabolite) is detectable in urine for 3-5 days
• Function: Used as an analgesic. following single use and for as long as 4
• Metabolism: Administered orally, with elimination weeks following chronic use.
dependent on it being metabolized by the liver. • Analysis: Immunoassays and Gas
• Toxic effects at high dosages: Chromatography/Mass Spectrometry.
o Acetaminophen – if present in serum at 300
mg/mL, 2 hrs. after ingestion induces hepatic Cocaine
toxicity. • Function: Used as a local anesthetic and at a higher
• Analysis: Immunoassays and high performance levels functions as a CNS stimulant that is
liquid chromatography. metabolized by Cholinesterase to form
benzoylecgonine, which is excreted by the kidney.
7. DRUGS OF ABUSE • Metabolism: Half-life 0.5-1 hour, with elimination
dependent on liver metabolism; benzoylecgonine
Amphetamines and Methamphetamine (half-life 4-7 hours) primary metabolite filtered by the
• CNS stimulants that block dopamine receptors in the kidney.
brain. • Benzoylecgonine is detectable in urine for 3 days
• Amphetamine metabolism occurs in the liver and following single use and for as long as 20 days
produces benzoic acid. following chronic use.
• Function: Used to treat narcolepsy and disorders • Intoxication: Cocaine overdose produces
that affect ability to focus; stimulants, provide sense hypertension, MI, or seizure. Cardiotoxicity can occur
of mental and physical well-being. and result in sudden death following cocaine use.
• Intoxication: Can produce severe depression, • Analysis: Immunoassays and Gas
respiratory difficulties, and episodes of paranoia. Chromatography/Mass Spectrometry
• Antidote: The antidote is forced acid diuresis.
• Analysis: Immunoassays and GLC Opiates
• Types:
Anabolic Steroids o Naturally Occurring
• Function: Used to increase muscle mass and athletic ▪ Opium, Morphine, Codeine
performance. o Chemically Modified
• Analysis: Gas or Liquid Chromatography ▪ Heroin, Dilaudid, Oxycodone
o Synthetic
▪ Demerol, Methadone, Darvon,
Talwin, Fentanyl
• Function: Narcotics used for their analgesic,
sedative, and anesthetic properties.
• Metabolism: Respiratory center depressant causing
respiratory acidosis.
• Analysis: Immunoassays and Gas
Chromatography/Mass Spectrometry

Phencyclidine (PCP)
• An abused anesthetic that is illegally used as a
hallucinogen.
• Function: Produces stimulant, depressant,
anesthetic, and hallucinogenic effects.
• Metabolism: Distributes in lipophilic tissue such as
the brain and adipose tissue; elimination dependent
on it being metabolized by the liver, with 10-15% of
the parent compound filtered by the kidney;
detectable in urine for as long as 7-30 days following
chronic use.
• Intoxication: This drug can produce violence,
seizures, respiratory depression, or death.
• Antidote: Treatment for PCP overdose is diazepam.
PCP is unmetabolized and excreted in the urine as
phencyclidine.
• Analysis: Immunoassays and GC/MS.

Tranquilizers
• Types:
o Barbiturates
▪ Phenobarbital – long acting
▪ Amobarbital – intermediate acting
▪ Secobarbital & Pentobarbital – short
acting
o Benzodiazepines
▪ Diazepam – valium
▪ Chlordiazepoxide – Librium
▪ Lorazepam – Ativan
• Function: Sedative hypnotics that produce
depression of the CNS.
o The barbiturates can be considered as
substances of abuse or as analgesics.
• Barbiturate intoxication results in cardiac arrest and
respiratory depression through its effect on the CNS.
• Analysis: Immunoassays and Gas-liquid
Chromatography

You might also like